Investments Test Bank Missed Q's 1

Réussis tes devoirs et examens dès maintenant avec Quizwiz!

Which of the following have been repurchased by the corporation? A) Unissued shares. B) Repurchased shares. C) Authorized shares. D) Treasury shares.

D) Treasury shares. Rationale The correct answer is "D." Unissued shares have never been held by investors to be repurchased. There is no such thing as "repurchased" shares. Authorized shares may be unissued or outstanding shares, but not necessarily Treasury shares (which are those the company has repurchased).

When an investment banker agrees to purchase an entire issue of securities from the issuing corporation and sell them to the general public, the agreement is called: A) A firm commitment. B) A best efforts agreement. C) A syndicated offering. D) A green shoe agreement.

A) A firm commitment. Rationale The correct answer is "A." A "best effort" agreement occurs when an underwriter agrees to sell what he or she can. A "syndicate" offering occurs when an underwriter forms a team of brokerage firms. A "green shoe" agreement is a standby commitment.

Which of the following elements of risk in mortgage-backed securities can be difficult to determine? I. Actual maturity is not known with certainty. II. Mortgage rates vary between the different investment pools. III. Actual cash flows are not known with certainty. IV. Government guarantees make the determination of an appropriate discount rate for calculating their present value difficult. A) I and III only. B) I and IV only. C) II and III only. D) II and IV only.

A) I and III only. Rationale The correct answer is "A." Lack of a definite maturity date and uncertain cash flows are the elements of risk in mortgage-backed securities.

Jocelyn Jane has come to you asking about investments that will produce steady income, provide relative safety of principal, and give her a tax advantage. What will you recommend that she consider adding to her portfolio? A) MBIA-backed municipal bonds. B) Zero-coupon U.S. Treasury bonds. C) Collateralized Mortgage Obligations. D) A mortgage REIT.

A) MBIA-backed municipal bonds. Rationale The correct answer is "A." The Municipal Bond Insurance Association provides protection against bond default (safe). Munis will provide steady income and they are tax advantaged. Treasury zeroes pay tax on accrued but unrealized income (phantom income.) CMOs, GNMAs, and Mortgage REITs all have volatility of payment and all run the risk of being paid off early if mortgagees decide to refinance their obligations.

Kimberly Thurman is a private investor who researches individual stock purchases thoroughly. She studies company annual reports and 10k reports, computes comparative financial ratios from the reports, and compares company financial information to industry statistics to find undervalued stocks. Kim believes in: A) The weak form of the efficient market hypothesis. B) The neglected firm effect. C) The random walk hypothesis. D) The semi-strong form of the efficient market hypothesis.

A) The weak form of the efficient market hypothesis. Rationale The correct answer is "A." Kimberly believes fundamental analysis will help her achieve above average market returns. The weak form of the EMH states that "the current price of a security reflects all historical information available on that security and does not reject fundamental analysis."

Your client has purchased stock with a margin position that required 50% initial margin and a 35% maintenance margin. The stock was originally valued at $23 per share when the transaction was undertaken and your client bought 1,000 shares. What stock price will trigger a margin call? A) $13.04 B) $17.69 C) $26.45 D) $32.86

B) $17.69 Rationale The correct answer is "B." Price = Loan ÷ (1 - MM) = (23 x. 50) ÷ (1 - .35) = 17.69

Which of the following statements are true regarding Dollar-Cost Averaging (DCA)? I. Investors put in the same dollar amount each month over time. II. An investor purchases the same number of shares each month over time. III. Dollar Cost Averaging lowers average cost per share over a period of time (assuming share price fluctuations). IV. An investor invests the same dollar amount each month to protect the investment from loss of capital. A) I and II only. B) I and III only. C) II and III only. D) II and IV only.

B) I and III only. Rationale The correct answer is "B." Choice "II" describes share averaging rather than DCA and Choice "IV" is untrue because DCA is not used to protect against capital loss.

The following set of newly issued debt instruments was purchased for a portfolio: - Treasury bond. - Zero-coupon bond. - Corporate bond. - Municipal bond. The respective maturities of these investments are approximately equivalent. Which one of the investments in the proceeding set would be subject to the greatest relative amount of price volatility if interest rates were to change quickly? A) Treasury bond. B) Zero-coupon bond. C) Corporate bond. D) Municipal bond.

B) Zero-coupon bond. Rationale The correct answer is "B." The smaller the coupon, the greater the volatility. Because the zero-coupon bond pays no payments, it is the smallest, thus the most volatile.

Which of these bonds initially immunizes a bond portfolio if the investors time horizon is 8 years? A) 20 year zero coupon B) Series of Tbills C) Coupon paying bond maturing in 8 years D) Coupon paying bond maturing in 10 years

D) Coupon paying bond maturing in 10 years Rationale The correct answer is "D." Option "A" - The term / duration is 20 years, which is well beyond the investors time horizon. Option "B" - The duration is too short (less than 12 month). Option "C" - The duration is something less than 8 which is too short. Option "D" - This is the only bond where the duration may equal the investor's time horizon, so it is the best answer.

If the market risk premium were to increase, the value of common stock (everything else being equal) would: A) NOT change because this does NOT affect stock values. B) Increase in order to compensate the investor for increased risk. C) Increase due to higher risk-free rates. D) Decrease in order to compensate the investor for increased risk.

D) Decrease in order to compensate the investor for increased risk. Rationale The correct answer is "D." A need for higher return to meet the onset of higher risk would drive the price of a security down (all other things being equal).

Your client owns a DGL Corporation convertible bond that has a coupon rate of 8% paid semiannually and matures in five years. Comparable debt yields 7% currently. The GGL bond is convertible into 22 shares of common stock. The current market price of the underlying stock is $52. What is the conversion value of this convertible bond? A) $925 B) $1,000 C) $1,042 D) $1,144

D) $1,144 Rationale The correct answer is "D." Use the formula for calculating conversion value of a bond. Keep in mind the conversion value may be different from the intrinsic value of the bond (which in equilibrium is the market price of the bond). CV = (PAR ÷ Cp) X Ps $22 x $52 Cp = conversion price PAR / Cp = conversion ratio Ps = Price of the stock

Which of the following is NOT a similarity between preferred stock and debt instruments? A) Preferred stock represents the same level of risk as debt to the buyer. B) Preferred stock pays a fixed income in its dividend. C) Preferreds are purchased for their income stream. D) Preferred stock is subject to interest rate and purchasing power risks.

A) Preferred stock represents the same level of risk as debt to the buyer. Rationale The correct answer is "A." Preferred stocks are riskier than debt due to the lack of a maturity date on preferred issues.

What does a stock dividend signal to investors? A) The firm is retaining capital growth related activities. B) The firm is preparing to spin off certain unprofitable divisions. C) The firm may be experiencing serious internal difficulties which is why it has no cash to put toward dividends. D) The firm is in transition as far as management goes and in these situations, stock is the preferred compensation rather than cash.

A) The firm is retaining capital growth related activities. Rationale The correct answer is "A." The stock dividend is often taken as a favorable signal. It is used for acquisition, increased R&D or occasionally to fend off takeovers. It is more common to see large amounts of debt taken on to make the firm seem less attractive if large amounts of cash are required.

Bristol-Buyers Company has a market price of $36.00 per share with earnings of $3.00 per share, a beta of 1.1 and a dividend of $1.20, which means a dividend payout ratio of 40%. Earnings for next year are projected to increase by 25%, and the retention ratio is projected to remain at 60%. Using the price/earnings multiplier, to what level might your client expect to see market prices move in a year? A) $39.60 B) $45.00 C) $50.40 D) $57.60

B) $45.00 Rationale The $36.00 per share price is divided by the $3.00 earnings per share resulting in a price/earnings multiplier of 12. The increase of earnings by 25% results in a projected earnings of $3.75 next year. This new earnings times the P/E multiplier of 12 (assuming the P/E ratio remains constant) results in a price of $45.00. The dividend information provided is unnecessary in answering the question.

Match the investment characteristic(s) listed below which describe(s) closed-end investment companies. A) Passive management of the portfolios. B) Shares of the fund are normally traded in major secondary markets. C) Both "A" and "B." D) Neither "A" nor "B."

B) Shares of the fund are normally traded in major secondary markets. Rationale The correct answer is "B." Close-end funds are traded on the secondary markets but are not passively managed.

Specific companies are researched and chosen as investments based on their outstanding investment possibilities by analysts who practice: A) The Dow theory analysis. B) Top-down analysis. C) Bottom-up analysis. D) Random Walk analysis.

C) Bottom-up analysis. Rationale The correct answer is "C." Bottom up analysts are looking for the next big, but as yet, undiscovered stock that will break onto the scene. Bottom up analysts start with the company, then the industry and finally the economic climate. Top-down starts with the economic climate, moves to the industry and then the company.

Walter owns a well diversified portfolio of common stock. He is concerned about a downturn in the market in the short term and prefers not to sell his portfolio of common stock. Which of the following strategies would you recommend? A) Sell a call option on the S&P 500. B) Buy a call option on the S&P 500. C) Buy a put option on the S&P 500. D) Sell a put option on the S&P 500.

C) Buy a put option on the S&P 500.. Rationale The correct answer is "C." Buying a put option on the S&P 500 will give Walter the most downside protection, in the event the market experiences a downturn. Selling a call option will not provide any downside protection; at most Walter will get to keep the premium. A strategy of selling a put option will lose value if the market declines in value.

The type of risk which measures the extent to which a firm uses debt securities and other forms of debt in its capital structure to finance is known as: A) Business risk. B) Systematic risk. C) Default risk. D) Financial risk.

D) Financial risk. Rationale The correct answer is "D." Financial risk has to do with the amount of leveraging or use of borrowed funds a firm utilizes to structure its investment and finance its assets.

David has $20,000 that is earmarked for a down payment on a house in two years. If David is in the 28% tax bracket, what should he invest the $20,000 in? A) A 4% tax free money market mutual fund. B) A 5.4% corporate bond. C) A well diversified growth mutual fund. D) An intermediate muni-bond fund paying 4.5%.

A) A 4% tax free money market mutual fund. Rationale The correct answer is "A." The taxable equivalent yield for the tax free money market fund is 5.56%. TEY = .04 / (1 - .28) TEY = .0556 The taxable equivalent yield is greater than the taxable corporate bond paying 5.4%. The mutual fund and intermediate muni-bond fund are not appropriate given the investor's time horizon.

Which one of the following statements best describes a firm commitment? A) The investment banker agrees to purchase the entire issue and resell the securities to the public. B) The SEC registration process for an underwriting is NOT as extensive as for an initial public offering. C) The investment banker agrees to sell a minimum number of shares before the offering closing date. D) The corporation issuing shares bears the risks associated with a failure to market the entire issue.

A) The investment banker agrees to purchase the entire issue and resell the securities to the public. Rationale The correct answer is "A." Choice "B" is an incorrect answer. Choice "C" is known as a best effort agreement. Choice "D" is not true. In a firm commitment, the issuing corporation does not bear the risks if an entire issue is not marketed.

If one of your clients has a profitable long position in an oranges futures contract and does nothing as the contract expires, what should she expect to occur? A) The oranges will be delivered to her. B) She will receive a substantial check as soon as the account is settled. C) Her contract will expire worthless unless she takes some action. D) Her broker will arrange for sale of the oranges in an appropriate market.

A) The oranges will be delivered to her. Rationale The correct answer is "A." Positions in futures contracts are closed by taking an equal and opposite position. One who is long on a contract at the expiration should expect delivery of the commodities at the stated contract price. It is the buyers responsibility, but in this case, we could say the broker was remiss in his or her duties!

Mutual fund XYZ has a beta of 1.5, standard deviation of 12% and a correlation to the S&P 500 of .80. How much return of fund XYZ is due to the S&P 500? A) 20%. B) 64%. C) 80%. D) 100%.

B) 64%. Rationale The correct answer is "B." Correlation is .80, therefore r-squared is .64 (R-squared = correlation coefficient squared). Therefore 64% of mutual fund's return is due to the S&P 500. Remember, r-squared measures the percentage of return due to the market.

Which of the following is/are characteristics of a municipal bond unit investment trust? I. Additional securities are NOT added to the trust. II. Shares may be sold at a premium or discount to net asset value. III. Shares are normally traded on the open market (exchanges.) IV. The portfolio is self-liquidating. A) I only. B) I and IV only. C) II and III only. D) II and IV only.

B) I and IV only. Rationale The correct answer is "B." Unit investments do not make additions to investments once the trust has been structured. Shares are not bought or sold after structuring and the portfolio is self-liquidating.

Your client believes the market may correct itself in the near future and has expressed an interest in using options to accomplish two goals in his existing portfolio. 1) He wants to increase his portfolio income, and 2) he wants to protect against any price drops in the underlying investments. To accomplish these goals, you would advise him to: A) Undertake naked call writing. B) Sell calls and sell puts. C) Sell calls and buy puts. D) Buy calls and sell puts.

C) Sell calls and buy puts. Rationale The correct answer is "C." Though no strike prices are specified, the buying of puts will allow your client to mitigate losses from occurring in the event of a drop in price. At the same time, the selling of calls will generate premium for the portfolio (additional income.) Should prices fall, they will expire worthless. If stock prices rise, the stock may be called away, but likely at a higher dollar value.

Bottom-up equity managers include: I. Group rotation managers. II. Value managers. III. Market timers. IV. Technicians. A) I only. B) II only. C) I and III only. D) II and IV only.

D) II and IV only. Rationale The correct answer is "D." Options "I" and "III" are both "top down" style managers.

An investor who would like to know how a portfolio manager performed relative to how the manager was expected to perform on a risk-adjusted basis would use which one of the following indicators? A) Sharpe index. B) Jensen index. C) Treynor index. D) Sharpe and Treynor.

Rationale The correct answer is "B." To compare a portfolio manager's performance to that of the market using Sharpe or Treynor models, one must calculate the results of both of these models on other portfolios or on a market being used as a benchmark, as well as the portfolio in question.

A put option with a strike price of $50 is selling for 3 1/2 when the market price of the underlying stock is $52. The intrinsic value of the put is: A) 0 B) 1 1/2 C) 2 D) -2

A) 0 Rationale The correct answer is "A." The intrinsic value of a put option is strike price minus stock price (50-52). If the answer arrived at is negative, then the intrinsic value is zero, and the value of the put is entirely related to the time component. In the case of a call, the intrinsic value is the stock price minus the strike price.

Robin purchased a mutual fund at NAV of $20.00 and sold it 8 months later at $21.00. During the time he owned the fund, he received a LTCG of $1.00/share and a qualified dividend distribution of $.75/share. He has a marginal tax rate of 30%. The tax on LTCG is 15%. What is his after-tax holding period return? A) 10.9% B) 11.5% C) 11.8% D) 12.3%

A) 10.9% Rationale Since this is a ST holding period, it's ordinary income at the marginal tax rate for the price increase. Since the dividend distribution is a qualified dividend, it receives capital gians tax treatment. HPR = (SP - PP +/- CF) x (1-TR) / PP [($21.00 - $20.00) x (1 - .30)] + [($1.00 + $.75) x (1 - .15)] / $20.00 Answer: 10.94% (remember to follow order of operations: Parenthesis, exponents, multiplication, division, addition, subtraction) Based on Subchapter M or pipeline theory, investment companies must payout at least 90% of their portfolio earnings. If a mutual fund sells a position they hold at a gain, it is passes the gain, like-kind, to its investors.

A convertible bond has the following terms: Maturity value = $1,000 Time to maturity = 20 years Coupon rate = 8% Call penalty = One year's interest Exercise price = $10 per share. Given this information, you will inform your client as to how many shares of stock that the bond may be converted. A) 100 shares B) 80 shares C) 50 shares D) 20 shares

A) 100 shares Rationale The correct answer is "A." The conversion ratio is used here (PAR/Conversion Price) to determine how many shares are available upon conversion. $1,000 / $10 = 100 shares.

John Henry has requested information regarding the risk involved in his overseas investments. He has given you the following information: Monrovia Investment: Standard deviation of 17.5%, Rate of return = 16.3% and Weight in portfolio of 60% Zimbabwe Investment: Standard deviation of 24.9%, Rate of return = 32.1% and Weight in portfolio of 40% Harry wants to know what the portfolio risk (as measured by portfolio deviation) is if the correlation coefficient of these two investments is negative .35 (-.35). A) 11.6% B) 9.7% C) 5.8% D) 1.36%

A) 11.6% Rationale The correct answer is "A." Use the formula for portfolio deviation to calculate the correct answer to this problem. S p = √ (.60)2(.175)2 + (.40)2(.249)2 + 2(.60)(.40)(.175)(.249)(-.35) S p = √ .0110 + .0099 - .0073 S p = √ .0136 S p = .1165

Jenny bought 250 shares of XYZ stock at $30 per share, with an initial margin of 60%. She paid 8% margin interest annually. One year later she sold all of the stock for $10,500. Jenny is in the 35% marginal tax bracket and itemizes her deductions. What is Jenny's holding period yield? A) 40% B) 61.33% C) 51.47% D) 34%

A) 40% Rationale Answer: A This question asks for holding period yield, not holding period return. To calculate the holding period yield, you simply subtract the purchase price from the selling price and divide by the purchase price (($10,500 / 250) - $30 )/ $30. Choice B is not correct because that is the calculation for holding period return. Choice C is not correct because that is the calculation for the after-tax holding period return. Choice D is not correct because that is the calculation for the after-tax holding period return without the margins.

You are faced with several fixed income investment options. Which of these bonds has the greatest reinvestment rate risk? A) A U.S. Treasury bond with an 11.625% coupon, due in five years with a price of $1,225.39 and a yield to maturity of 6.3%. B) A U. S. Treasury strip bond (zero-coupon) due in five years with a price of $735.12 and a yield to maturity of 6.25%. C) A corporate B-rated bond with a 9.75% coupon, due in five years with a price of $1,038.18 and a yield to maturity of 8.79%. D) A corporate zero coupon bond due in 5 years with a price of $750 and a yield to maturity of 5.9%.

A) A U.S. Treasury bond with an 11.625% coupon, due in five years with a price of $1,225.39 and a yield to maturity of 6.3%. Rationale The correct answer is "A." This is due to the high coupon and lack of similar rates currently.

An investor who searches for stocks selling at a low price to earnings (P/E) ratio believes that: A) Anomalies to the Efficient Market Hypothesis exist. B) The strong form of the Efficient Market Hypothesis is valid. C) Such stocks have low betas. D) The semi-strong form of the Efficient Market Hypothesis is valid.

A) Anomalies to the Efficient Market Hypothesis exist. Rationale The correct answer is "A." The low P/E ratio stocks are an anomaly to the EMH. Choice "B" is incorrect and the strong form of EMH is often thought to be invalid because it presumes markets are completely efficient and historical, public and private information will not help you achieve above average market returns. Choice "C" is incorrect as the stocks could have either high or low betas. Choice "D" is incorrect because the evaluation of P/E ratios is fundamental analysis, and the semi-strong theory rejects fundamental analysis (and technical analysis).

Over the previous several months, Mathew Arkins, CFP®, has presented senior executives at West-Ark Financial Planning with numerous research reports indicating that several of the recommendations West-Ark has been making to their clients are no longer in favor with the analysts providing the research reports. Despite this apparent change in heart by the analysts, West-Ark has yet to adjust its advice to clients. Which behavioral bias is most evident for West-Ark? A) Belief perseverance. B) Herd mentality. C) Hindsight bias. D) Overconfidence.

A) Belief perseverance. Rationale Answer: A Belief perseverance is evident when people are unlikely to change their views given new information. B is incorrect. Herd mentality is the process of buying what and when others are buying and selling. C is incorrect. Hindsight bias is a form of overconfidence related to an investor's belief that they had predicted an event that, in fact, they did not predict. D is incorrect. Overconfidence suggests that investors overestimate their ability to successfully predict future market events.

Which of the following reveals the relationship of a given security's movement relative to that of the market? A) Beta. B) Correlation coefficient. C) Covariance. D) Standard deviation.

A) Beta. Rationale The correct answer is "A." Correlation coefficient and covariance measure two stocks movements relative to one another. Standard deviation measures a security's performance relative to expectations of performance. Alpha reveals the level of over or underperformance of the security relative to market expectations. (tb 3171)

Carolyn Gay is considering the purchase of 2 bonds for her portfolio. She has specific investment horizons and wishes to know if these issues match her needs based on duration. Please advise her of the duration of each bond. Bond Market Value Coupon Rate Years to Maturity Interest Rate A $551.83 6% 20 12% B $1,098.18 9% 20 8% A) Bond A: 9.5 years / Bond B: 10.4 years. B) Bond A: 10.4 years / Bond B: 8.7 years. C) Bond A: 12.9 years / Bond B: 9.5 years. D) Bond A: 15.2 years / Bond B: 17.8 years.

A) Bond A: 9.5 years / Bond B: 10.4 years. Rationale The correct answer is "A." Use the duration formula to calculate the duration of these two specific investments. D = (1 + y) - (1 + y) + t (c - y) y c[(1 + y)t - 1] + y D = (1.12) - 1.12 + 20 (.06 - .12) .12 .06 [(1.12)20 - 1] + .12 D = 9.33 - -.0800 .6388 D = 9.4582 Bond A YTM: N = 20 i = ? PV = <551.83> PMT = 60 FV = 1,000 YTM = 12 D = (1.08) - 1.08 + 20 (.09 - .08) .08 .09 [(1.08)20 - 1] + .08 D = 13.5 - 1.28 .4095 D = 10.3742

Bond A has a 6% annual coupon and is due in 2 years. Its value in today's market is $900. Bond B has a 10% annual coupon and is due in 4 years. It is priced to yield 12%. Bond C is a zero-coupon bond priced to yield 11% in 8 years. Assuming the duration of Bond A is 1.94 years, which of the following statements about the effect of a 1% decline in interest rates is true? A) Bond C, having a longer duration that Bond A, would have a larger percent increase in price than Bond A. B) The percent change in price of a bond is independent of the duration of a bond. C) It is NOT possible to determine the percent change in price of Bond A versus Bond C because the duration of Bond C is NOT given. D) Bond A would have a greater percent change in price than Bond C because it has a shorter duration.

A) Bond C, having a longer duration that Bond A, would have a larger percent increase in price than Bond A. Rationale The correct answer is "A." In this instance, there's no need to do any calculations. They're already (inherently) done. Because it gives the duration of Bond A (directly) and Bond C (indirectly as zero coupon bonds always have a duration equal to their maturity), the longer duration bond (Bond C) will have the greatest sensitivity to interest rate changes (because of the longer duration). Bond B is ignored simply because it's not found in the answers as a choice. Thus, answer A is correct. Answer B is not correct because the percent price change is dependent on duration. Answer C is not correct because duration is given (indirectly by giving the maturity and the fact it's a zero). Answer D is incorrect simply because it's not true of duration. If you choose to calculate: Multiply the change of rate by the duration to arrive at the percent change in price (see formula sheet for percent change formula). For each percent of change, multiply by the duration to arrive at the total percent change. Then calculate that percentage into a price change. **This may exceed the 1.5 minutes per question suggested on the exam.

What is one implication of the efficient market hypothesis? A) Consistently superior performance is rare. B) The weak form suggests that security prices do NOT adjust to new information. C) The random walk hypothesis is invalidated. D) Anomalies are perceptions but do NOT actually exist.

A) Consistently superior performance is rare. Rationale The correct answer is "A." One cannot consistently earn abnormal returns. Over time, these superior returns will be reversed and overall gain is consistent with the general market.

Which of the following is the better indicator for evaluating a bond's sensitivity to interest rate risk? A) Duration, because it measures interest rate and reinvestment rate changes. B) Covariance, because it measures the relationship between interest rate changes and bond price changes. C) Yield to maturity, because it takes capital gains, losses and periodic coupon payments into account. D) Weighted-average maturity, because it recognizes the full term of the bond.

A) Duration, because it measures interest rate and reinvestment rate changes. Rationale The correct answer is "A." Duration is the time remaining when a security's discounted future cash flow remains at risk. This is an accurate measure of a bond's sensitivity to interest rate risk.

Which of the following is NOT a premium factor that would be considered part of the nominal rate of interest? A) Economic premium. B) Default premium. C) Liquidity premium. D) Risk free rate of interest.

A) Economic premium. Rationale The correct answer is "A." There is no such thing as an economic premium. All of the other premiums added to the risk free rate equal the nominal (or stated) rate.

Which one of the following is an advantage of equity REITs over mortgage REITs? A) Equity REITs can participate in the appreciation of the underlying properties. B) Equity REITs participate in the capital gains of the mortgages, whereas mortgage REITs receive only the coupon payments. C) Equity REITs retain the right to the potential appreciation of a property, but mortgage REITs retain the right to only the property's rental income. D) Equity REITs have the right to repossess the underlying property if the mortgage REIT fails to make its mortgage payments.

A) Equity REITs can participate in the appreciation of the underlying properties. Rationale The correct answer is "A." Option "B" describes mortgage REITs, not equity REITs. Option "C" is incorrect as mortgage REITs have nothing to do with "rental income." Option "D" is an incorrect statement.

Your client will only invest in securities backed by the full faith, credit and taxing power of the U.S. government. Which of the following should she consider for her portfolio? A) Government National Mortgage Association Certificates (Ginnie Maes). B) Student Loan Marketing Association Notes (Sallie Maes). C) Tennessee Valley Authority (TVA) Securities. D) Federal Home Loan Mortgage Association Debentures (Freddie Macs).

A) Government National Mortgage Association Certificates (Ginnie Maes). Rationale The correct answer is "A." Options "B," "C" and "D" are all agency debt, and though there has never been a default on a U.S. government agency debt, it is not backed by the "full faith, credit and taxing power of the U.S. government" as are GNMAs and Treasury issues.

Which of the following best describes the investment characteristics of a high-quality long-term municipal bond? A) High inflation risk; low default risk. B) Low inflation risk; high market risk. C) Low inflation risk; low default risk. D) High inflation risk; high market risk.

A) High inflation risk; low default risk. Rationale The correct answer is "A." The rate of the high-quality municipal bond will be low, thus exposing it to potentially higher levels of inflation risk. These same instruments are very safe and represent low default risk.

During the peak of the economic cycle, which of the following should one undertake? I. Sell debt instruments II. Begin allocations to cash positions III. Buy debt instruments IV. Sell gold and real assets A) I and II only. B) II and III only. C) III and IV only. D) I and IV only.

A) I and II only. Rationale The correct answer is "A." At the peak of the cycle, as the economy has reached full steam, it is an excellent time to sell not buy fixed (and generally lower return) instruments. It is also an excellent time to begin appropriations to cash in preparation for opportunities that may arise. Since inflation is still on (and rising after the peak), it often proves to be a good time to acquire metals rather than sell them.

Your client owns the following two corporate bonds: Bond Rating Coupon Maturity ABC AA 5.25% 16 RST BBB 8.50% 9 Which of the following statements are true about the relationship between the bond prices and bond features? (Consider each statement with respect to only the single feature stated; do not attempt to integrate the impact of all features simultaneously). I. The lower coupon makes ABC's bond more volatile than RST's bond. II. The longer maturity makes ABC's bond more volatile than RST's bond. III. The higher coupon makes RST's bond more volatile than ABC's bond. IV. RST's lower rating does not make its volatility higher or lower than ABC's volatility. A) I and II only. B) I and IV only. C) II and III only. D) II and IV only.

A) I and II only. Rationale The correct answer is "A." Choice "I" - The lower the coupon, the more volatile the bond. Choice "II" - The longer the maturity, the more volatile the bond. The greater the volatility, the greater the risk to the investor.

Which of the following statements about preferred stock are true? I. Its market fluctuations are greater than the long-term bond market fluctuations. II. It is more risky than debt. III. Its dividends are recomputed quarterly. IV. It has no interest rate risk because it is a stock and not a bond. A) I and II only. B) I and IV only. C) II and III only. D) II and IV only.

A) I and II only. Rationale The correct answer is "A." Option "III" - Dividends are set at issue by the Board as a percent of par value and do not change. Option "IV" - Changes in interest rates directly impact preferred stock, and there is no relief on preferred stock because most cannot be held to maturity (as most are issued without a maturity date). Preferred stock is more risky than bonds because bonds are a legal obligation and have a higher priority in bankruptcy proceedings.

John Risotto has a cash need at the end of nine years. Which of the following investments best meets this need and serves to immunize the portfolio initially? I. An 11-year maturity coupon bond. II. A 9-year maturity coupon Treasury note. III. A series of Treasury bills. A) I only. B) II and III only. C) II only. D) I and II only.

A) I only. Rationale The correct answer is "A." The process of portfolio immunization entails not maturity of a security, but its duration. Duration is based on coupon rate. The larger the coupon payment, the shorter the duration. This being the case, a bond generally pays higher interest than a note, and a note pays higher than short-term Treasury bills. Given this information, one could reasonably expect a shorter duration (than time to maturity), while receiving better immunization from the bond.

Which of the following statement(s) regarding bond swaps is/are true? I. A substitution swap is designed to take advantage of anticipated and potential yield differentials between bonds that are similar with regard to coupons, rating, maturities, and industry. II. Rate anticipation swaps utilize forecasts of general interest rate changes. III. The yield pickup swap is designed to alter the cash flow of the portfolio by exchanging similar bonds having different coupon rates. IV. The tax swap is made to substitute current yield in place of capital gains. A) I, II and III only. B) I and III only. C) II and IV only. D) IV only.

A) I, II and III only. Rationale The correct answer is "A." All statements are correct except for IV. The tax swap replaces bonds with offsetting capital gains and losses.

Which of the following represent the minimum information needed to calculate the weighted average rate of return for a portfolio? I. Current market price of each security. II. Price paid for each security. III. Number of shares of each security. IV. Total portfolio value. V. Percent return of each security. A) I, III and V only. B) II, III and IV only. C) III, IV and V only. D) I, IV and V only.

A) I, III and V only. Rationale The correct answer is "A." Current market price times number of shares gives the total value of each investment. These can be summed to calculate the total portfolio value. Then, the total value of each investment is divided by the total portfolio value and multiplied by the percent return of each security to calculate the weighted return. These weighted returns added together give the weighted average portfolio rate of return.

An investor may use options on debt instruments to protect against: A) Interest rate risk. B) Reinvestment rate risk. C) Default risk. D) Call risk.

A) Interest rate risk. Rationale The correct answer is "A." Put options that lock in the price at which the security may be sold may be used to protect an investor from a drop in bond prices caused by rising interest rates.

Which of the following terms would be used to describe a municipal bond issued with a restricted revenue base? A) Limited general obligation bonds. B) Limited revenue bonds. C) Full faith and credit bonds. D) Revenue bonds.

A) Limited general obligation bonds. Rationale The correct answer is "A." The limited general obligation bond is a bond issued by an entity that has some ability to levy taxes to support itself (for example, a school district). However, this ability is limited when compared to that of the general taxing power of the state.

The primary reason for using a ladder bond strategy is to: A) Lower overall interest rate risk. B) Achieve greater capital gains as the yield curve changes shape. C) Avoid the "wash sale" rule. D) Immunize the bond portfolio.

A) Lower overall interest rate risk. Rationale The correct answer is "A." The ladder bond strategy staggers maturities and in doing so, reduces the exposure to interest rate risk. "B" is incorrect because longer term maturities experience the biggest percentage increase when interest rates decrease. Laddering bonds requires purchasing short and intermediate term bonds, along with long term bonds. "C" is incorrect because laddering bonds to avoid the wash sale rule is not the primary objective, reducing interest rate risk is the primary objective. "D" is incorrect because bond immunization suggests the portfolio has eliminated interest rate and reinvestment rate risk. Laddering bonds does not eliminate either interest rate or reinvestment rate risk, however it does reduce interest rate risk.

The form of technical analysis that utilizes Advances and Declines (also known as Breadth of the Market) as an indicator is known as: A) Price Indicator. B) Volume Indicator. C) Market Indicator. D) Charting Indicator.

A) Price Indicator. Rationale The correct answer is "A." Advances and declines deal with price. Volume indicates the number of shares traded. Market indicators deal with directions of the market and related averages. Charts are used as indicators and in some instances, do not use price but rather movements.

Which of the following requires registration under disclosure rules with the Securities and Exchange Commission? A) Sale of an entire issue of securities in an IPO. B) Sale of securities in a single block to a public pension fund. C) Sale of an entire issue of securities to a single investor. D) Sale of securities in a single block to a publicly-traded mutual fund.

A) Sale of an entire issue of securities in an IPO. Rationale The correct answer is "A." All answers except "A" constitute private placement.

Which one of the following factors would be the strongest indication that interest rates might rise? A) Selling of dollar-denominated assets by foreign investors. B) Decreasing United States government deficits. C) Decreasing rates of inflation. D) Weak credit demand by the private sector of the United States economy.

A) Selling of dollar-denominated assets by foreign investors. Rationale The correct answer is "A." Foreigners selling dollar-denominated assets are preparing to take advantage of higher rates by increasing their liquidity. The rest signal a decrease in rates.

John was the editor of his campus conservative newspaper. He was a member of the National Guard and was on the university's precision rifle shooting team. He was a vocal supporter of U.S. military actions in Iraq and Afghanistan. You knew John was interested in becoming a financial analyst, but you have lost touch with him after you both graduated from college. Which of the following statements is more likely to be true? 1. John is employed as an analyst with Second National Securities. 2. John is employed as an analyst with Second National Securities and is a member of the National Rifle Association (NRA). A) Statement 1 B) Statement 2 C) They are equally likely to be true. D) The answer can't be determined with the information provided.

A) Statement 1 Rationale Answer: A However likely it is that John works for Second National Securities, the likelihood that he works for Second National and also is a member of the NRA cannot exceed the stand-alone probability of working for Second National. Furthermore, they could only be equally probable if each and every analyst employed by Second National is also a member of the NRA, which is itself an improbable event. Representativeness takes you towards Statement 2 as your mind views it as "completing the story." However, the hard cold logic of mathematical probability confirms that Statement 1 is indeed more likely to be correct.

Bob Conrad's investment portfolio consists of several types of stocks, bonds, and money market instruments. The portfolio has an overall standard deviation of 12%, a beta of 1.06, and a total return for the year of 11%. Bob is considering adding one of two alternative investments to his portfolio. Stock A has a standard deviation of 13%, a beta of .87, and a correlation coefficient with the portfolio of .6. Stock B has a standard deviation of 11%, a beta of .97, and a correlation coefficient of .95. Which stock should Bob consider adding to his portfolio, and why? A) Stock A, because it has a lower correlation coefficient. B) Stock A, because it has a lower beta than that of the portfolio. C) Stock B, because it has a lower standard deviation than that of the portfolio. D) Stock B, because it has a higher correlation coefficient.

A) Stock A, because it has a lower correlation coefficient. Rationale The correct answer is "A." In the process of adding new investments to a portfolio, the lowest correlation coefficient makes the best addition. Closest to negative one (-1) is always best.

An investor who rebalances her portfolio frequently to take advantage of perceived opportunities in other market sectors is using which one of the following types of asset allocation? A) Tactical. B) Strategic. C) Passive. D) Hybrid.

A) Tactical. Rationale The correct answer is "A." Strategic asset allocation is concerned with allocating the wealth of a client among various asset classes, consistent with the clients' investment objectives, time horizons and risk preferences. Tactical asset allocation is concerned with shifting wealth between asset classes to take advantage of expected price level changes (timing) arising from broad movements in the business cycle.

Which of the following statements is correct with regard to the use of an arbitration clause in an investment advisory agreement? A) The SEC and FINRA require arbitration if voluntary negotiation fails. B) The SEC requires that such a clause be contained in any investment advisory agreement. C) The FINRA requires that such a clause stipulate that arbitration must be conducted by non-industry organizations. D) The clause must allow state regulations to take precedence over federal regulation.

A) The SEC and FINRA require arbitration if voluntary negotiation fails. Rationale The correct answer is "A ." Both SEC and FINRA call for voluntary negotiations first. Barring success with this level of contact both SEC and FINRA require arbitration.

What is one reason a company may call bonds that were previously issued? A) The bonds are currently selling at a premium. B) The bonds are currently selling at a discount. C) The company expects interest rates to decrease. D) The bonds are selling at par.

A) The bonds are currently selling at a premium. Rationale The correct answer is "A." If the bonds are selling at a premium, then interest rates have decreased since the bonds were issued. The company would be motivated to retire the higher yield bonds and issue new bonds at lower market interest rates. A discount bond would indicate that interest rates of increased and the bond is paying a lower rate than current market interest rates.

In an after-dinner conversation, your neighbor states that Hot-Flow, Inc. must certainly be a good investment now that the stock has fallen from its recent high of $80 per share. The company currently trades for $65 per share. You ask your neighbor if she has any other information on which to base her buy recommendation. "Not really," she replies, "but if the stock was $80 per share last month, surely it will return to that level in the near future. After all," she continues, "how much can things change in just a few months of time?" Your neighbor's attitude is best described as: A) anchoring. B) hindsight bias. C) regret avoidance. D) representativeness.

A) anchoring. Rationale Answer: A Anchoring results in buying securities that have fallen in value because it "must" get back up to that recent high. B is incorrect. Hindsight bias is a form of overconfidence related to an investor's belief that they had predicted an event that, in fact, they did not predict. C is incorrect. Regret avoidance, also known as the disposition effect, causes investors to take action (or inaction) in hopes of minimizing any regret. D is incorrect. Representativeness is thinking that a good company is a good investment without regard to an analysis of the investment.

Sue Todd began purchasing BLT, Inc. mutual fund shares several years ago. She has followed a dollar-cost averaging approach by investing $1,000 each year for 5 years. The following describes Sue's purchases: Year 1 - $1,000 investment at $120 per share Year 2 - $1,000 investment at $100 per share Year 3 - $1,000 investment at $118 per share Year 4 - $1,000 investment at $97 share Year 5 - $1,000 investment at $130 per share What is Sue's average cost per share? A) $100 B) $111.58 C) $113 D) $118.23

B) $111.58 Rationale The correct answer is "B." Calculate the number of shares purchased over time (44.8 shares) and divide this figure into the total amount invested over that time ($5,000). The result is an average share price of $111.58 per share. Year 1 - $1,000 investment at $120 per share = 8.33 shares Year 2 - $1,000 investment at $100 per share = 10 shares Year 3 - $1,000 investment at $118 per share = 8.47 shares Year 4 - $1,000 investment at $97 share = 10.30 shares Year 5 - $1,000 investment at $130 per share = 7.69 shares $5,000 / 44.79 = 111.63 (give or take for rounding)

Roger Vincent has a portfolio with 500 shares of Appel with a current value of $27 per share, a return of 15.3%, and a beta of 1.35. He also has 250 shares of Bobbel with a return of 10.7%, a .83 beta, and currently priced at $62 per share. Finally he owns 750 shares of Cybert priced at $54 per share, with a 1.1 beta and a 12.9% return. What is the weighted average portfolio beta of Roger's portfolio? A) .97 B) 1.09 C) 1.14 D) 1.21

B) 1.09 Rationale The correct answer is "B." We carry out a weighted average beta calculation with total portfolio value of $69,500 and each stock as a percent of that total, times its individual beta, all added together to provide the weighted average portfolio beta. Appel 500 X $27 = $13,500 (y1) Bobbel 250 X $62 = $15,500 (y2) Cybert 750 X $54 = $40,500 (y3) 1.35 (x1), .83 (x2), & 1.1 (x3) STAT, LIN. x bar= 1.0933

Which of the following best describes the fees charged based on the average daily fund assets and used principally to meet marketing expenses are called: A) Front-end load. B) 12b-1 fees. C) Back-end load. D) Deferred sales charge.

B) 12b-1 fees. Rationale The correct answer is "B." The above statement describes 12b-1 fees.

What is the portfolio deviation of a portfolio invested 60% in stock "A" with a 15% return and a deviation of 17.5%, and the balance in stock "B" with an 18% return and a 16.75% deviation. There is a .29 correlation between the two securities. A) 16.2% B) 14.0% C) 13.05% D) 4.69%

B) 14.0% Rationale The correct answer is "B." COV = .175 x .1675 x .29 = .0085 s p = √(.60)2(.175)2 + (.40)2(.1675)2 + 2(.60)(.40)(.0085) s p = √.011 + .0045 + .0041 s p = √.0196 s p = .1399

An analysis of the monthly returns for the past year of a mutual fund portfolio consisting of two funds revealed the following statistics: Fund A total return = 18% Fund A Standard deviation = 23% Fund A Percentage of portfolio = 35% Fund B total return = 11% Fund B Standard deviation = 16% Fund B Percentage of portfolio = 65% The Correlation Coefficient ("R") between the two funds equals .25. What is the standard deviation of the portfolio? A) 13.16% B) 14.66% C) 18.45% D) 19.50%

B) 14.66% Rationale The correct answer is "B." The information in [ ] is the COV formula. compute that prior to multiplying 2(.35)(.65)COVij s p = √(.35)2(.23)2 + (.65)2(.16)2 + 2(.35)(.65)[(.23)(.16)(.25)] s p = √.0065 + .0108 + .0042 s p = √.0215 s p = .1466

An investor buys a share of stock for $50. At the end of the first year, he purchases a second share for $55. At the end of the second year, the stock is worth $62 per share and the investor sells both shares. (The investor received a cash dividend of $2 per share each year.) What is the time-weighted return on this investment? A) 13.6% B) 15.2% C) 16.5% D) 18.3%

B) 15.2% Rationale The correct answer is "B." CFo = <50>, CFj = 2, CFj = 64 (62 + 2) then solve for IRR. Remember, time-weighted return is only concerned about the security's cash flow, not the investors.

An investor in the 30% marginal tax bracket purchased a bond for $980, received $75 in interest, and then sold the bond for $950 after holding it for seven months. The LTCG rate is 15%. What are the pre-tax and post-tax holding period returns? A) 4.6%, 4.6% B) 4.6%, 3.2% C) 11%, 4.6% D) 11%, 3.2%

B) 4.6%, 3.2% Rationale Pre-tax Holding Period Return = ($950 - $980 + $75) / $980 = 4.6% Since the holding period is ST, use the marginal tax rate: After-tax Holding Period Return = [($950 - $980 + $75) x (1 - .30)] / $980 = 3.2% Answer: 4.6%, 3.2%

You are faced with several fixed income investment options. Which of these bonds has the greatest interest rate risk? A) A U.S. Treasury bond with an 11.625% coupon, due in five years with a price of $1,225.39 and a yield to maturity of 6.3%. B) A U.S. Treasury strip bond (zero-coupon) due in five years with a price of $735.12 and a yield to maturity of 6.25%. C) A corporate B-rated bond with a 9.75% coupon, due in five years with a price of $1,038.18 and a yield to maturity of 8.79%. D) A U.S. T-bill selling for $950 due in six months.

B) A U.S. Treasury strip bond (zero-coupon) due in five years with a price of $735.12 and a yield to maturity of 6.25%. Rationale The correct answer is "B." With the term being equal, the bond with the lowest coupon will have the biggest duration. The bigger the duration, the price more sensitive the bond is to interest rate changes. Bond B has the lowest coupon, zero.

The lowest bond rating that can still be considered investment grade debt is: A) A B) Baa C) BB D) Caa

B) Baa Rationale The correct answer is "B." Baa is the lowest bond rating in Moody's Rating System, while BBB is the lowest investment grade in the S&P bond rating system.

To be on a corporation's books as a holder-of-record (and thus have a right to the next dividend payment), the investor must purchase stock: A) Before the declaration date. B) Before the ex-dividend date. C) Between the ex-dividend date and the record date. D) Three days before the payment date.

B) Before the ex-dividend date. Rationale The correct answer is "B." As of the ex-dividend date, the stock sells without right to collect the next expected dividend due.

Developing cash flow projections and valuations for real estate can be difficult due to: A) A lack of comparable figures for other properties in the area. B) Changes in demographic and economic variables. C) Different financing methods amongst prospective purchasers. D) A lack of standardized methods for objectively evaluating an investment in a market that is considered inefficient.

B) Changes in demographic and economic variables. Rationale The correct answer is "B." Cash flow projections and comparable equity capitalization rates are easily obtained for a valid comparison. The difficulty is one of the unpredictability of changes in economics and demographics which directly impact values. Real estate valuation models such as one using net operating income, adjust for variations in real estate financing.

A mutual fund investor who is looking for the opportunity to buy investments at a discount, so as to capture a greater portion of any capital gains, would probably decide to invest in a(n): A) Open-end fund. B) Closed-end fund. C) Unit investment trust. D) Exchange Traded Fund (ETF).

B) Closed-end fund. Rationale The correct answer is "B." Closed-end funds generally sell at either a premium or a discount to par value. When purchased at a discount, they afford investors an opportunity to realize up-side capital appreciation.

Your client holds a diversified equity portfolio, and has asked for your opinion on what is the most important factor to consider as he prepares to add additional equity securities. You tell him: A) Standard deviation of the new additions. B) Correlation of the new securities to the portfolio. C) Coefficient of variation of the new securities. D) Beta of the new securities to the existing portfolio.

B) Correlation of the new securities to the portfolio. Rationale The correct answer is "B." Correlation of securities is always the strongest determinant as to what should be added to a portfolio.

The date on which the current dividend no longer accompanies the stock is: A) Holder of record date. B) Ex-dividend date. C) Trade-free date. D) Declaration date.

B) Ex-dividend date. Rationale The correct answer is "B." Ex. - dividend date.

Margin accounts involve security transactions performed using some amount of capital borrowed from the brokerage firm as well as some of the investor's own capital. The entity that establishes the initial margin requirement is the: A) Securities and Exchange Commission. B) Federal Reserve. C) National Association of Securities Dealers. D) Brokerage firm with which an investor is dealing.

B) Federal Reserve. Rationale The correct answer is "B." The Federal Reserve sets margin requirements for all security transactions.

James avidly follows several television shows that discuss securities, including, typically, making buy/sell recommendations of individual stocks. James almost always adjusts his portfolio as per these recommendations. James is best described as exhibiting which behavioral bias? A) Belief perseverance. B) Herd mentality. C) Hindsight bias. D) Overconfidence.

B) Herd mentality. Rationale Answer: B Herd mentality is the process of buying and selling what and when others are buying and selling. A is incorrect. Belief perseverance is similar to anchoring in that people are unlikely to change their views given new information. C is incorrect. Hindsight bias is a form of overconfidence related to an investor's belief that they had predicted an event that, in fact, they did not predict. D is incorrect. Overconfidence suggests that investors overestimate their ability to successfully predict future market events.

Which of the following are results of security regulations on investments? I. The Federal Reserve Bank places a limit on the amount of credit that can be used to transact the acquisition of securities. II. Most new issues must be registered with the SEC. III. Investors are insured against investment losses by SIPC for any securities registered with the SEC. IV. Officers of an issuing firm may be held liable for material omissions or misstatements in the prospectus. A) I, II and III only. B) I, II and IV only. C) II, III, and IV only. D) I and III only.

B) I, II and IV only. Rationale The correct answer is "B." Statement "III" - SIPC insures investors against losses due to bankruptcy or insolvency of brokerage firms. There is no protection against investment losses.

Which of the statements below correctly describes the potential price volatility of the following five bonds? Bond A: A-rated; 7% coupon; matures in 10 years. Bond B: A-rated; 6% coupon; matures in 12 years. Bond C: A-rated; 7% coupon; matures in 12 years. Bond D: A-rated; 5% coupon; matures in 15 years. Bond E: A-rated; 6% coupon; matures in 15 years. I. Bond A has greater potential for price fluctuations than Bond C. II. Bond D has greater potential for price fluctuations than Bond E. III. Bond C has greater potential for price fluctuations than Bond B. IV. Bond D has greater potential for price fluctuations than Bond B. A) I and II only. B) II and IV only. C) III and IV only. D) I, II, and IV only.

B) II and IV only. Rationale The correct answer is "B." Choice "I" (Bonds A and C) have the same coupon, but Bond A has a shorter maturity therefore less volatility. Choice "II" (Bonds D and E) both mature in 15 years, but Bond D's lower coupon makes it more volatile. Choice "III" (Bonds C and B) both have the same maturity, but Bond B's lower coupon means it is more volatile. Choice "IV" (Bonds B and D) is a true statement due to both coupon and maturity.

In the mutual fund industry, 12b-1 fees are charged as part of: I. Fund management fees. II. Distribution fees. III. Commissions for sales. IV. Legal fees and expenses. A) I only. B) II only. C) I and III only. D) I, II and IV only.

B) II only. Rationale The correct answer is "B." 12b1 fees are used for marketing and distribution costs. All other costs, such as legal, accounting and analysis are paid through management fees. Commissions are paid using either a front load or a back load.

Which of the following can be eliminated using a "buy and hold" strategy with regard to fixed income securities? A) Future value risk. B) Interest rate risk. C) Stand alone risk. D) Reinvestment rate risk.

B) Interest rate risk. Rationale The correct answer is "B." The price changes when interest rates change but if you don't sell the bond (buy and hold) then the price change doesn't really matter. You still get the $1,000 par value at maturity. Option "A" - Future value risk does not exist as a term. Option "C" - Stand alone risk refers to single assets ownership. Option "D" will still require the investor to reinvest interest paid, thus not eliminating such risk.

Sylvia has a two assets in her portfolio, asset A and asset B. Asset A has a standard deviation of 40% and asset B has a standard deviation of 20%. 50% of her portfolio is invested in asset A and 50% is invested in asset B. The correlation for asset A and asset B is .90. What is the standard deviation of her portfolio? A) Greater than 30%. B) Less than 30%. C) Equal to 30%. D) Not enough information to determine.

B) Less than 30%. Rationale The correct answer is "B." It's not necessary to use the standard deviation of a two asset portfolio formula to answer this question. Since there's a 50/50 weighting for each asset, simply take a simple average of the standard deviations (.40 + .20) / 2 = .30. Since the correlation is less than 1, the standard deviation for the portfolio will be less than the simple average. If correlation was equal to 1, then the standard deviation would be equal to 30%.

A yield curve normally is upward sloping because: A) Long-term rates must be higher to compensate for higher expected future tax rates. B) Long-term bonds are, by their nature, more risky than short-term bonds. C) Higher long-term rates reflect inflationary expectations. D) Short-term bonds have a lower level of event risk.

B) Long-term bonds are, by their nature, more risky than short-term bonds. Rationale The correct answer is "B." Short-term rates are lower because of the lower risk associated with them. The longer an investment ties up an investor's capital, the higher the rate must be to offset this risk. Inflation expectations may be lower in the future, resulting in a downward (inverted) sloping yield curve.

Often, municipal bonds are insured. One group which insures them is the: A) Municipal Insurance Group. B) Municipal Bond Insurance Association. C) Federal Insurance Guarantee Corporation. D) Resolution Trust Corporation.

B) Municipal Bond Insurance Association. Rationale The correct answer is "B." Another group which insures municipal bonds is the American Municipal Bond Assurance

A stock that has produced superior earnings and rates of return but has gone mostly unnoticed by securities analysts and is often considered underpriced is said to benefit from the: A) P/E effect. B) Neglected firm effect. C) Small firm effect. D) Low price/sales effect.

B) Neglected firm effect. Rationale The correct answer is "B." The neglected firm effect is one of the market anomalies. This anomaly is said to exist because the security in question is allowed greater potential for movement as a result of the lack of scrutiny by analysts.

LeAnn Wallace, CFP®, is discussing with her colleague an article she recently read in a behavioral finance academic journal. She comments that while behavioral finance is quite interesting, it is also confusing. She finds it difficult to separate one behavioral trait from another, and difficult to separate the consequences of the various traits. She starts to diagram the traits and their consequences. Her first two entries read: Behavioral bias: Consequence: Hindsight bias Belief that one has predicted an event that, in fact, they did not predict Cognitive dissonance Minimizing or forgetting past losses Exaggerating past gains Which of the following behavioral biases is closest to the two identified in Ms. Wallace's diagram? A) Belief perseverance. B) Overconfidence. C) Regret avoidance. D) Representativeness.

B) Overconfidence. Rationale Answer: B Hindsight bias and cognitive dissonance are each a type of overconfidence. A is incorrect. Belief perseverance is similar to anchoring in that people are unlikely to change their views given new information. C is incorrect. Regret avoidance, also known as the disposition effect, causes investors to take action (or inaction) in hopes of minimizing any regret. D is incorrect. Representativeness is thinking that a good company is a good investment without regard to an analysis of the investment.

If the risk/return performance of a stock lies above the Security Market Line, the stock is said to have a: A) Positive correlation coefficient. B) Positive alpha. C) Positive expected return. D) Positive covariance.

B) Positive alpha. Rationale The correct answer is "B." Performance of a stock below the SML is a negative alpha. Again, the Jensen formula can be used for this calculation.

Municipal bonds that are backed by the income from specific projects are known as: A) Income bonds. B) Revenue bonds. C) General obligation bonds. D) Debenture bonds.

B) Revenue bonds. Rationale The correct answer is "B." Revenue generated from the project, such as a toll to pay for the bridge, that is used to repay such municipal obligations are known as revenue bonds. The other municipal bonds, general obligation bonds, are backed by the taxing power of the issuing body.

A child is 8 years old and the parents want to invest today for the child's education. The parents have AGI of $140,000. Which investment vehicle would you recommend? A) Series EE savings bonds. B) S&P 500 index fund. C) Laddered CDs D) Money market mutual fund.

B) S&P 500 index fund. Rationale The correct answer is "B." The S&P 500 index fund is the best answer because the time horizon is long term (10 years). The parents are currently phased-out of the interest income tax exclusion benefit on the series EE savings bonds. The CDs and money market mutual fund are too conservative.

Jasmine has a large paper profit in her Amalgamated Corporation shares, currently at $46 per share. She is happy with the stock, but realizes that a good thing CANNOT go on forever. She bought the stock so inexpensively that she is not worried about the downside. If she is willing to sell at $50, what strategy could you recommend to her? A) Buy $50 call options. B) Sell $50 call options. C) Buy $50 put options. D) Sell $50 put options.

B) Sell $50 call options. Rationale The answer is "B." She gains the premium from selling the call, and if the price rises, at or above the strike price of $50, her stock will be called away at $50. "C" would be a good choice, but she is not worried about the downside risk.

The Federal Reserve Board is expected to sell large quantities of Treasury securities in the near future. What impact will these sales likely have on stock prices? A) Stock prices will decrease because the dividend growth rate of stocks will increase. B) Stock prices will decrease because the required rate of return for investors will increase. Rationale The correct answer is "B." The sale of Treasury securities results in a reduction of cash in the market place, thus a decrease in supply causing an increase in demand. This will lead to an increase in the cost of money and a lessening of funds for investment, thus a reduction in stock prices. C) Stock prices will increase because interest rates will decrease as investors compete to purchase the Treasury securities. D) Stock prices will increase because the growth rate in dividends and earnings will increase.

B) Stock prices will decrease because the required rate of return for investors will increase. Rationale The correct answer is "B." The sale of Treasury securities results in a reduction of cash in the market place, thus a decrease in supply causing an increase in demand. This will lead to an increase in the cost of money and a lessening of funds for investment, thus a reduction in stock prices.

Which method of portfolio evaluation allows the comparison of a portfolio manager's performance to that of the over-all market using just one calculation? A) The Treynor Model. B) The Jensen Model. C) The APT Model. D) The Sharpe Model.

B) The Jensen Model. Rationale The correct answer is "B." Only Options "A," "B" and "D" are models used to examine portfolio manager's performance. Treynor and Sharpe require that one calculate the performance of the market to make a valid comparison.

When evaluating the return of two investment managers, the performance measurement approach generally used is the: A) Internal rate of return. B) Time-weighted return. C) Value-weighted return. D) Dollar-weighted return.

B) Time-weighted return. Rationale The correct answer is "B." Time-weighted rate of return is a compounded rate of return calculation that allows for a more accurate comparison based on the security's cash flows. Internal rate of return is a compunded rate of return, but we need to be more specific whether to consider the security's cash flows or the investor's cash flow.

In comparing the performance of two mutual funds over the previous five-year period, you note that the annual returns of the funds are quite similar year-to-year. You also note that one of the funds has a portfolio turnover that averages 12 times the value of the portfolio (i.e., the dollar amount of trades the fund made for the year is 12 times the average dollar value of the portfolio) For one year, turnover was as high as 26. The comparison fund's portfolio turnover never exceeded 0.8 and averaged 0.4 for the five year period. From a behavioral finance point of view, the high turnover fund's management most likely exhibits: A) anchoring. B) overconfidence. C) regret avoidance. D) representativeness.

B) overconfidence. Rationale Answer: B Overconfidence leads to overtrading. A is incorrect. Anchoring results in buying securities that have fallen in value because it "must" get back up to that recent high. C is incorrect. Regret avoidance, also known as the disposition effect, causes investors to take action (or inaction) in hopes of minimizing any regret. D is incorrect. Representativeness is thinking that a good company is a good investment without regard to an analysis of the investment.

Of the following indexes, which is the only one that uses the geometric average to compute its daily value? A) NASDAQ Index. B) Wilshire 5000 Index. C) Value Line Average. D) Dow Jones Industrial Average.

C) Value Line Average. Rationale The correct answer is "C." The NASDAQ, the NYSE Composite, and the Wilshire all use value weighted average, while the Dow Jones Industrial is a simple price weighted average. Only Value Line uses the geometric average.

You purchase one put contract and pay a $3 premium that allows you to sell the stock at $50. The stock is currently trading at $53. What is the intrinsic value of your situation? A) -$5 B) -$2 C) $0 D) $2

C) $0 Rationale The correct answer is "C." Intrinsic Value of Put = Strike Price - Stock Price, therefore IV = $50 - $53= $- 3, however intrinsic value cannot be negative.

Your client, Dennis and Daughter, Inc. (often referred to as DAD by the owners) is a C corporation with gross revenues of $3,000,000 for the past four years. The net earnings to the firm for the most recent fiscal year were $120,000. There are two shareholders, Dennis and his daughter, Denise. They have recently had an outside consultant perform a valuation of the company using the capitalization method and a .10 capitalization rate. Based on this information, Dennis and Denise have decided to consider a buy-sell agreement. Using the above information, answer the following question. The value of the firm, as established by the outside consultant, is likely to be closest to: A) $833,333 B) $1,000,000 C) $1,200,000 D) $1,333,400

C) $1,200,000 Rationale The correct answer is "C." Divide the capitalization rate factor into the net earnings to arrive at the correct response. $120,000 ÷ .10 = $1,200,000

My margin requirements are 50% initial margin and 25% maintenance margin. I purchase a total of 200 shares at $100 per share using full margin amount for the 200 share purchase. Shortly thereafter, share prices fall to $50 per share. What will my margin call be? A) $1,000 B) $1,500 C) $2,500 D) $5,000

C) $2,500 Rationale The correct answer is "C." Required equity: $50 x .25 = 12.50 per share Actual equity: $50 - $50 = 0 (current price- loan amount) To meet required equity: $12.50 per share x 200 shares = $2,500

A convertible bond your client is interested in buying has a 8% coupon rate, a conversion ratio for 100 shares, while market rates on similar issues are currently 12%. It pays interest twice a year, has a par value of $1,000 with a maturity of 20 years. The current stock price is $8.50 per share. What price should your client expect to pay? A) $1,125 B) $1,000 C) $850 D) $699

C) $850 Rationale The correct answer is "C." The calculated value as a debt instrument here is $699.07, but when the conversion value puts the rate at $850, you can be certain that the bond is selling in the market place at its convertible value of $850, not its debt value of $699. CV = (1,000 ÷ CP) x Ps $850 OR N = 20 x 2 = 40 i = 12 ÷ 2 = 6 PV = ? PMT = (.08 x 1,000) ÷ 2 = 40 FV = 1,000 PV = 699.07 The conversion price is not given; however, (1,000 ÷ CP) is the conversion ratio, which is given (100 shares). Therefore: CV = 100 shares x Ps CV = 100 x 8.50 CV = $850

Barbara Reed owns an LMN, Inc. bond with a par value of $1,000. LMN is a AA-rated bond that matures in 7 years. Barbara receives $55 of interest income from LMN semiannually. Comparable debt, i.e., is AA-rated, 7-year maturity, yields 12%. The bond's duration is 5 years. What is the intrinsic value of the bond? A) $703.36 B) $880.80 C) $953.53 D) $954.36

C) $953.53 Rationale The correct answer is "C." The intrinsic value of a bond is its calculated present value. N = 7 x 2 i = 12 ÷ 2 PV = ? PMT = 55 ($110 ÷ 2) FV = 1,000

Roger Vincent has a portfolio with 500 shares of Appel with a current value of $27 per share, a return of 15.3%, and a beta of 1.35. He also has 250 shares of Bobbel with a return of 10.7%, a .83 beta, and currently priced at $62 per share. Finally he owns 750 shares of Cybert priced at $54 per share, with a 1.1 beta and a 12.9% return. What is the weighted average portfolio return of Roger's portfolio? A) 14.67% B) 13.33% C) 12.88% D) 11.29%

C) 12.88% Rationale The correct answer is "C." We carry out a weighted average return calculation with total portfolio value of $69,500 and each stock as a percent of that total, times its individual return, all added together to provide the weighted average portfolio return. Appel 500 X $27 = $13,500 (Y1) Bobbel 250 X $62 = $15,500 (Y2) Cybert 750 X $54 = $40,500 (Y3) Total $69,500 15.3 (x1), 10.7 (x2), & 12.9 (x3) Data (input x & y variables) Stat, 1-V, x bar = 12.875

A client has bought a stock for $40 per share. At the end of the first year, she purchases another share at $43 per share. At the end of the second year with the share price of $48, she sells her shares. Along the way, at the end of each year, she received a $2 per share dividend. What is the time-weighted return on her investment? A) 9.53% B) 13.5% C) 14.3% D) 16.6%

C) 14.3% Rationale The correct answer is "C." This is simply an uneven cash flow problem. CF0 = <$40> CF1 = $2 CF2 = $50 IRR = 14.33% Note: Since this is a time weighted return, we are only concerned about the security's cash flow. Therefore, we ignore the second purchase at $43 per share.

Which one of the following statements correctly matches a technical indicator to the information it provides to signal a bear market? A) Odd lot theory indicates that the ratio of odd lot purchases to odd lot sales has been falling. B) Dow theory confirmation after the fact not predictive indicates that there is a decline in both the Dow Jones Industrial Average and Dow Jones Utility Average. C) A moving average chart indicates that actual prices have dropped through the moving 200 day average line. D) Barron's Confidence Index indicates that the yield differential between municipal bonds and corporate bonds is increasing.

C) A moving average chart indicates that actual prices have dropped through the moving 200 day average line. Rationale The correct answer is "C." Choice "A" - Odd lot purchase levels indicate the number of small investors in the market. Odd lot theory says that small investors are always wrong. If odd lot purchases are falling relative to odd lot sales, it indicates the little guy thinks the market will fall. Since the little guy is always wrong, this would indicate a rally is coming, not a bear market. Choice "B" - The Dow Theory deals in three levels of market activity over time. Choice "D" - Barron's does not have a confidence index.

A rise in the price of the Japanese Yen in relation to the U.S. Dollar results in: A) A devaluation of the Yen. B) Excess reserves in the U.S. current account. C) A revaluation of the Yen. D) A negative balance of payments.

C) A revaluation of the Yen. Rationale The correct answer is "C." Were the Yen to fall in value against the dollar, this would constitute a devaluation, but when it costs more dollars to buy a Yen, this is considered an appreciation or revaluation of the Yen.

The Federal Reserve is currently tightening the money supply. As the treasurer and CFO of your company, which of the following best describes the hedge position that you should undertake and the reason for taking it to protect your company's long-term bond inventory. A) A short position to hedge against increases in bond prices. B) A long position to hedge against increases in bond prices. C) A short position to hedge against decreases in bond prices. D) A long position to hedge against decreases in bond prices.

C) A short position to hedge against decreases in bond prices. Rationale The correct answer is "C." You own a long position on the bonds. A tightening of money will cause a rise in the interest rates, thus exposing your bonds to a loss in value when bond prices decrease as a result. You should undertake a short (sell) position in interest rate futures to protect your position.

American Depository Receipts (ADRs): A) Eliminate currency exchange rate risk. B) Eliminate currency restrictions of foreign countries. C) Allow U.S. investors to buy foreign country stock denominated in dollars. D) Are generally exempt from capital gains taxes in the United States.

C) Allow U.S. investors to buy foreign country stock denominated in dollars. Rationale The correct answer is "C." ADRs do not eliminate currency exchange rate risk (Choice "A") or currency restrictions of foreign countries (Choice "B") and tax will be paid on capital gains (Choice "D").

An increasing inflation rate can have a negative effect on the value of common stock and bonds due to: A) The U.S. Treasury having to increase the supply of money to try to reduce inflation. B) A reduction in interest rates, which will reduce the value of stocks. C) An increase in the investor's required rate of return. D) An increase in the international transfer of funds.

C) An increase in the investor's required rate of return. Rationale The correct answer is "C." Because of the upward pressure of inflation on interest rates, the demand by investors for increased returns puts pressure on companies to perform at a time when the increased cost of borrowing may make that impossible. The company will be seen as less valuable, stock may be sold, and stock prices will fall as a result.

These bonds are considered to be owned by whoever possesses them: A) Serial bonds. B) Registered bonds. C) Bearer bonds. D) Reset bonds.

C) Bearer bonds. Rationale The correct answer is "C." Serial bonds are issued in series and mature in series. Registered bonds are paid interest based on to whom the bonds are registered. Bearer bonds pay interest to the holder of the bond. Interest rates can be reset on Reset bonds, and the U.S. government issues Treasury bonds; and both are registered.

In his search for better ways to invest, your client, John Galt, hears of the barbell bond strategy. He asks you what this investment strategy is, and if you would recommend it for him. You advise John that for the barbell bond strategy to be undertaken correctly, the portfolio must be structured as follows: A) Bonds with maturities staggered every few years apart are purchased for inclusion in the portfolio. B) Bonds are purchased for the portfolio based on their duration, not on their maturity. C) Bonds for the portfolio are purchased with both long and short periods of time to maturity, with little in between. D) Bonds purchased for the portfolio are either very high risk, or they are very safe and secure offerings.

C) Bonds for the portfolio are purchased with both long and short periods of time to maturity, with little in between. Rationale The correct answer is "C." Barbell bond strategy has to do with long and short maturities in the portfolio . Option "A" is the bond-ladder strategy, and Option "B" describes immunization.

Match the investment characteristic(s) listed below which describe(s) a unit investment trust. A) Passive management of the portfolios. B) Self-liquidating investments usually holding bonds. C) Both "A" and "B." D) Neither "A" nor "B."

C) Both "A" and "B." Rationale The correct answer is "C." Both statements are correct because a UIT typically holds municipal bonds until maturity. UITs can also own equities.

A client with a large, well-diversified common stock portfolio expresses concern about a possible market decline. However, he/she does NOT want to incur the cost of selling a portion of their holdings NOR the risk of mistiming the market. A possible strategy for him/her would be: A) Buy an index call option. B) Sell an index call option. C) Buy an index put option. D) Sell an index put option.

C) Buy an index put option. Rationale The correct answer is "C." A put option index that closely mirrors the client's portfolio will allow for minimization of loss in the event of market decline.

Which one of the following types of investor benefits most from the tax advantage of preferred stocks? A) Government. B) Individual. C) Corporate. D) Mutual funds.

C) Corporate. Rationale The correct answer is "C." The corporate dividend-received deductions are based on ownership. TCJA of 2017 updated the amounts. If a corporation owns 20% or less, the have a DRD of 50%. if 20% or more (and less than 80%) of the corporation paying the dividend is owned by the company receiving the dividends, then up to 65% of the dividend is tax free. If ownership is greater than 80% (affiliated corporations) the DRD is 100%.

Your clients, Dan & Mary both work for Terra Corporation. They have asked you, as their personal financial planner, to explain to them exactly what their human resources department was referring to in a session last week when they discussed "out-of-the-money" positions in the stock option investment plan they have at work. Of the following, which would meet that description given the current $51 per share market price of the stock? A) Dan holds 100 shares of company stock which he purchased through his broker at $56 per share. B) A portion of Mary Jo's shares were given to her at the program's inception 18 months ago and are currently exercisable at $47 per share. C) Dan's most recent award has an exercise price of $55 per share. D) Mary Jo has 225 shares of Terra, awarded with an exercise price of $51 per share.

C) Dan's most recent award has an exercise price of $55 per share. Rationale The correct answer is "C." Option "A" is out-of-the-money but has nothing to do with his option through his work. Option "B" is in-the-money. Option "D" is at-the-money, meaning price of exercise and stock price are the same. Only Option "C" is out-of-the-money and addresses our client's concerns.

Given the following diversified mutual fund performance data, which fund had the best risk-adjusted performance if the risk-free rate of return is 5.7%? Fund A: Average rate of return = .0782, Standard deviation of annual return = .0760 and Beta = 0.950 Fund B: Average annual return = .1287, Standard deviation of annual return = .1575 and Beta = 1.250 Fund C: Average annual return = .1034, Standard deviation of annual return = .1874 and Beta = 0.857 Fund D: Average annual return = .0750, Standard deviation of annual return = .0810 and Beta = 0.300 A) Fund B, because the annual return is highest. B) Fund C, because the Sharpe ratio is lowest. C) Fund D, because the Treynor ratio is highest. D) Fund A, because the Treynor ratio is lowest.

C) Fund D, because the Treynor ratio is highest. Rationale The correct answer is "C." If a fund is diversified, use the Treynor model and the result there is arrived at by dividing the return by the beta. In this case, fund D has the highest risk adjusted rate of return. Treynor = [(rp - rf) / (Bp)]. In this case, the result is (.0750 - 0.057) ÷ .3000 = .06.

Herkimer Green has just inquired about purchasing callable bonds. You explain what this means, and you also explain the negative aspects for investors with regard to callable issues. These include: I. The uncertainty about the amount of payments to be made to the bondholders. II. The price risk for the investor. III. The inflation risk to the bondholder. IV. The reinvestment risk faced by the bond investor. V. The investor's liquidity risk. A) I only. B) II and III only. C) I and IV only. D) II, IV and V only.

C) I and IV only. Rationale The correct answer is "C." Price risk is not a callable bond concern; in fact, often when called a premium is paid. Inflation risk, where the purchasing power of the bond is affected might seem like a good choice, but because the bond is being held to call in this case, it is not a concern and liquidity risk is eliminated. When the bond is called, an investor does not need to worry about selling the bond.

Which of the following are factors to consider when investing in a mutual fund? I. The size of the fund. II. The amount of time until a distribution is made. III. The amount of time the current portfolio manager has managed the fund. IV. The availability of a third-party analysis of the fund. A) I and III only. B) II and IV only. C) I, II and III only. D) I, III and IV only.

C) I, II and III only. Rationale The correct answer is "C." If an investor can find out I, II and III, he or she will not likely require IV. Though it may reinforce the investor's findings, the third party analysis is unnecessary at that point.

Which of the following are true statements about the Capital Asset Pricing Model (CAPM)? I. The Security Market Line (SML) by itself does NOT determine the optimal portfolio for an investor. II. Beta is used as a measure of risk on the Security Market Line (SML). III. The required return is beta times the market return. IV. As investors replace risk-free assets with risky assets, the required return will rise. A) I and III only. B) II and IV only. C) I, II, and IV only. D) II, III, and IV only.

C) I, II, and IV only. Rationale The correct answer is "C." Choice "III" - The required rate of return is determined by adding the risk premium (which is the market rate of return minus the risk free rate times the beta) to the risk free rate of return.

Ben Robinson owns two stocks in equal dollar amounts with the following characteristics: Stock Expected Returns Standard Deviation Beta ABC 9.5% 13% 0.93 XYZ 12.0% 18% 1.12 The correlation of returns between the two stocks is -.89. Which of the following statements are true? I. The standard deviation of the portfolio is the average of the standard deviations of the two stocks. II. The standard deviation of the portfolio is less than the standard deviation of either of the two stocks. III. A negative correlation achieves a better portfolio diversification than would a positive correlation. IV. The diversification achieved by a combination of ABC and XYZ in a portfolio depends more on the average portfolio beta than on the correlation. A) I and III only. B) I and IV only. C) II and III only. D) II and IV only.

C) II and III only. Rationale The correct answer is "C." We are combining two stocks into a portfolio. Therefore we know the return will be the weighted average return of each of the stocks and the standard deviation of the portfolio will be less than the weighted average return of each of the stocks since the correlation is less than 1. Statement I is incorrect per the statement above which eliminates answers A and B. We have already narrowed the problem down to 50/50. Statement II has to be correct since only C and D are possible answers. This is proven in the comment above. Statement III is a correct statement since a correlation less than 1 means the stocks actually move opposite each other to some degree. This provides our portfolio with enhanced diversification over having two stocks with positive correlation. Statement IV is incorrect since the covariance / correlation is the most important factor regarding diversification. So even if you didn't know III was true you could narrow it down by eliminating I and IV as choices.

Which of the following statements about U.S. Treasury securities are correct? I. The smallest denomination of Treasury bills is $5,000. II. Treasury bills are issued at a discount. III. Treasury bonds with 30-year maturities have default risk. IV. The maximum maturity on Treasury notes upon issue is 10 years. A) I and II only. B) II and III only. C) II and IV only. D) I, II and IV only.

C) II and IV only. Rationale The correct answer is "C." Choice "I" - The smallest increment on a T-Bill is $100. Choice "III" - There is NO default risk associated with U.S. government Treasury issues.

Which of the following investment vehicles are most appropriate for an emergency fund for a family with $12,000-a-year discretionary income? I. Balanced mutual fund. II. Line of credit. III. Money market mutual funds. IV. Laddered CDs set to mature every 6 months. A) I and II only. B) II and IV only. C) III and IV only. D) I, II and III only.

C) III and IV only. Rationale The correct answer is "C." Though a line of credit might make a reasonably good emergency fund, the question asks for "investment vehicles" (which make Options "III" and "IV" correct.)

In order to determine whether a stock is overvalued or undervalued, a planner would use which of the following formulas? A) Security market line formula. B) Expected rate of return formula. C) Intrinsic value formula. D) Required rate of return formula.

C) Intrinsic value formula. Rationale The correct answer is "C." Options "A", "B", "C" and "D" are real formulas. Options "A" and "D" are the CAPM formula, and "B" is used to calculate what one should expect as an interest rate of return, not a price.

The bond investment strategy of "riding the yield curve" involves: A) Investing equal amounts in short-term and long-term bonds. B) Investing equal amounts in each of several maturity periods. C) Investing either short-term or long-term to take advantage of anticipated interest rate changes. D) Selling bonds with unrealized losses and replacing them with similar bonds.

C) Investing either short-term or long-term to take advantage of anticipated interest rate changes. Rationale The correct answer is "C." Riding the yield curve refers to the purchase of debt instruments in anticipation of fluctuations in the rates of return on both long and short-term instruments. Rising rates of interest require repositioning a portfolio in advance of the rise in order to avoid significant price drops. These moves are based on anticipated changes in the yield curve.

Your client, Bill Brown, is an investor in ONLY dividend paying stocks. He buys them in time to catch the dividend and then sells them. You have a stock that you have researched, Gamma Globe, and it generally pays very high dividends relative to its price. What will you advise Bill to do in regard to this stock? A) Wait until the stock goes ex-dividend to make the purchase, and sell immediately thereafter. B) Get in on Gamma Globe's action for the long haul, that is "buy and hold." C) Purchase the stock to own it by record date, and sell when the price has rebounded. D) It doesn't matter when Bill buys it, but once purchased, Gamma Globe should be held only in preparation for almost certain subsequent price increases.

C) Purchase the stock to own it by record date, and sell when the price has rebounded. Rationale The correct answer is "C." It is the only answer that matches the client's express strategy. With the current T+2 settlement time frame, the client would need to purchase 2 business days prior to record date, or the business day prior to ex-date. Buy and hold is not what the client wants, and ex-dividend dates are too late to get in on the dividend action which the client seeks.

The Securities Act of 1933 is best summarized by the following statement: A) Requires the registration and provides for regulation of investment advisors. B) The Act regulates securities in the secondary markets. C) Regulates both initial public offerings and subsequent secondary offerings by a public company. D) Established the organized securities exchanges.

C) Regulates both initial public offerings and subsequent secondary offerings by a public company. Rationale The correct answer is "C." Option "A" - The Investment Advisors Act of 1940 regulates advisors. Option "B" is incorrect because the Act of 1934 regulates the secondary market. Option "C" - The Act of 1933 regulates both IPOs and secondary offerings. Option "D" - The organized exchanges and previously issued securities are governed by the Securities and Exchange Act of 1934.

A yield curve can be described as a curve that: A) Depicts the current yield on government debt. B) Slopes upward as the years to maturity increase. C) Shows the term structure of interest rates on government debt. D) Offers greater potential yield as the maturity approaches the present.

C) Shows the term structure of interest rates on government debt. Rationale The correct answer is "C." The yield curve demonstrates graphically the relationship between long-term and short-term government debt.

As a measure for risk, the Capital Market Line (CML) uses the: A) Risk free rate of return. B) Beta of the market. C) Standard deviation of the market. D) Portfolio weighted beta.

C) Standard deviation of the market. Rationale The correct answer is "C." The CML (Capital Market Line) uses standard deviation, while the SML (Security Market Line) uses the beta as its "risk" measurement.

Hannah Latham, a client of yours, recently bought an orange grove. She has asked you about locking in the future price of her crop to assure adequate funds to meet expenses for the coming year. You explain both forward contracts and futures contracts to her and advise that she take the following position: A) Structure a forward contract with the bank and the juice maker. B) Take a futures contract position short the commodity and long the contract. C) Take a futures contract position long the commodity and short the contract. D) Set aside enough money to cover expenses and hope for the best.

C) Take a futures contract position long the commodity and short the contract. Rationale The correct answer is "C." A forward contract (though very specific) requires a buyer and a seller, and the grower may not know yet to whom the oranges will finally go. The farming and produce business are far too risky to be left to chance. And since she has the oranges in the trees, she should be long the commodity and sell a contract (or "short" the contract.)

The value of the convertible bond as a debt instrument does not depend on which of the following? A) The bond's coupon. B) Current interest rates. C) The conversion price of the bond. D) The term of the bond.

C) The conversion price of the bond. Rationale The correct answer is "C." The value of the bond as a debt instrument is considered separately from its convertibility and is calculated using the bond formula or the present value methodology on the calculator. However, should the conversion value be greater than the "debt value" of the bond, it will sell for the higher price.

Which of the following is an attribute of mortgage REITs? A) They receive income from the rental or lease of real estate properties. B) They provide more opportunity for capital gains than equity REITs. C) They receive monthly income from investing in real estate loans. D) They frequently invest in commercial properties.

C) They receive monthly income from investing in real estate loans. Rationale The correct answer is "C." Options "A", "B", and "D" all describe equity REITs. Mortgage REITs are more volatile than equity REITs.

Mark Wallens, CFP®, is reviewing a client's choices relating to refinancing the mortgage on the client's primary residence. Recent turmoil in the residential mortgage market has introduced significant uncertainty into the market. Rates over the past few weeks have varied significantly, even from one hour to the next. Mark's client was a subprime borrower when she took out an option adjustable rate mortgage (option ARM) and would have to pay a prepayment penalty of $6,000 to refinance. Mark's analysis suggests that there is a 40% chance the client could save $20,000 (in present value terms) with a new mortgage from First State Bank but there is a 60% chance she would be no better off with the new mortgage ($0 in present value terms). Mark also contacts the client's current lender and that bank is willing to renegotiate the current loan waiving the prepayment penalty. The new mortgage from the existing lender would save the client $1,000 (in present value terms) The client is having difficulty choosing which option to pursue. She tells Mr. Wallens that while she recognizes that the new mortgage from First State Bank has a higher expected value, the possibility of spending $6,000 and receiving no benefit is very unattractive. The client's dilemma is most consistent with: A) anchoring. B) cognitive dissonance. C) loss aversion. D) risk aversion.

C) loss aversion. Rationale Answer: C Many individuals, indeed most individuals, exhibit loss aversion. Loss aversion notes that people more strongly prefer to avoid losses than to seek gains. Loss aversion was identified by Amos Tversky and Daniel Kahneman. Kahneman received the Nobel Prize in Economics in 2002 for his work on prospect theory and loss aversion. Note - loss aversion is not included in the pre-study materials for investments, but I'm hoping this extremely important behavioral finance concept is included in the larger set of investment planning materials used by The Dalton Review. A is incorrect. Anchoring represents the investor's inability to objectively review and analyze new information. B is incorrect. Cognitive dissonance is a form of overconfidence because an investor's memory of past performance is better than the actual results. D is incorrect. Risk aversion is not considered to be a behavioral bias. Rather, risk aversion is an assumption of traditional financial analysis based on the precepts of rational, utility-maximizing economic theory.

An old Wall Street saying is, "Cut your losses and let your profits run." However, investors often do the opposite. It seems that people are reluctant to admit they made a mistake in purchasing a stock that subsequently performs poorly. This behavior is most consistent with: A) anchoring. B) herd mentality. C) regret avoidance. D) representativeness.

C) regret avoidance. Rationale Answer: C Regret avoidance (also known as the disposition effect) leads investors to take action or to refuse to act in hopes of minimizing any regret over their actions or inactions. In investments, it leads people to sell winners too soon and to hold on to losers too long. A is incorrect. Anchoring represents the investor's inability to objectively review and analyze new information. B is incorrect. Herd mentality is the process of buying what and when others are buying and selling. D is incorrect. Representativeness is thinking that a good company is a good investment without regard to an analysis of the investment.

The Chesapeake Bay apartment complex contains 60 one-bedroom apartments renting for $650 per month. In addition, the complex generates $625 per month from laundry, parking, and vending machines. Vacancy and collection losses have averaged 8% of Potential Gross Income (PGI) and are expected to continue at about the same rate in the future. Annual expenses totaling $117,000 include: Property taxes = $2,000 Property management = $7,000 Interest expense = $72,000 Swimming pool = $5,000 Professional fees = $8,000 Other expenses = $23,000 There is a monthly mortgage payment of $10,000 per month. Out of the $10,000 mortgage, $6,000 is interest expense and $4,000 is repayment of principal. Assuming a capitalization rate of 9%, what is the market value of the Chesapeake Bay complex? A) $1,941,422 B) $2,884,140 C) $3,560,667 D) $4,360,667

D) $4,360,667 Rationale The correct answer is "D." Gross rental receipts ($650 x 60 x 12) = $468,000 plus non-rental income ($625 x 12) = $7,500 equals potential gross income (PGI) ($468,000 + $7,500) = $475,500. PGI minus vacancy and collection losses [$475,500 - (.08 x $475,500)] = $437,460 equals Effective Gross Income (EGI). EGI minus expenses equals net income $437,460 - $117,000 = $320,460. Next, determine net operating income by adding interest and depreciation expense back to net income. NOI = $320,460 + $72,000 interest + $0 depreciation = $392,460. Market value = $392,460 ÷ .09 = $4,360,667

Haley Mills has been a client of yours for quite some time. She recently unearthed some share certificates that her grandmother had left to her a few years ago. These shares that Grandma Mills bought were from an Internet start up. The timing was perfect, because the firm was about to undertake another stock offering and Haley had preemptive rights. Of the firm's initial 1,700,000 share offering, Grandma Mills had invested enough to buy 10,000 shares at $11 per share. The new offering was an 850,000 share offering at $87 per share. If Haley fully exercised her preemptive rights, how much total cash would she pay for the shares in this new offering? A) $55,000 B) $217,500 C) $385,750 D) $435,000

D) $435,000 Rationale The correct answer is "D." The 10,000 share purchase of the 1,700,000 share initial offering was .59% (10,000 ÷ 1,700,000). That amount relative to this new offering of 850,000 shares was equivalent to 5,000 (850,0000 X .59%) shares if all preemptive rights were exercised. This 5,000 shares times the $87 offering price means that to fully exercise this right, Haley would require $435,000.

What is the geometric rate of return for a stock that has experienced the following prices over a four-year period? Year 1 = $20 Year 2 = $32 Year 3 = $24 Year 4 = $28 A) 17.23% B) 15.78% C) 13.82% D) 11.88%

D) 11.88% Rationale The correct answer is "D." There are many ways to solve this, but here is the quickest: N=3 i=? PV=<20> PMT=0 FV=28 Assume you paid $20 for the stock today and three years later, it is trading at $28.

Short-term debt securities issued by corporations in the open market usually less than 270 days maturity to avoid the necessity of SEC registration, are known as: A) Banker's Acceptance. B) Repurchase Agreements. C) Forward Contracts. D) Commercial Paper.

D) Commercial Paper. Rationale The correct answer is "D." Banker's acceptance are paper traded between banks such as letters of credit to facilitate international trade. Repurchase agreements are money market securities sold at a discount with an agreement to purchase them back at a higher price later on. Forward contracts are agreements to transact in the future.

Holly bought a stock at the minimum margin, when the stock was trading at $10. The stock paid quarterly dividends of $.25. Holly held the stock for one year and sold the stock when it was trading at $11. What was Holly's holding period return? A) 10%. B) 20%. C) 30% D) 40%.

D) 40%. Rationale The correct answer is "D." The first key to this question is knowing that the "minimum margin" is 50%, which is established by the Federal Reserve. So, Holly is required to pay $10 x .50 = $5 in cash and borrow the other $5 per share to make the investment. The question does not reference any margin interest, so it's excluded from the calculation. The second key to this problem is that the Purchase Price in the numerator reflects both the equity contribution of $5 per share and the $5 per share that must be repaid to the broker. The Purchase Price in the denominator only needs to reflect the $5 in equity paid. HPR = (SP - PP +/- CF) ÷ PP HPR = ($11 - $10 + ($.25 x 4) ÷ ($10 x .5) HPR = 40%

Your client's federal marginal tax rate is 36%, and the state marginal rate is 7%. The client does not itemize deductions on his federal return and is considering investing in a municipal bond which yields 5% issued in his state of residence. What is the taxable equivalent yield? A) 3.20% B) 4.65% C) 5.38% D) 8.77%

D) 8.77% Rationale The correct answer is "D." Use the tax equivalent yield formula of the taxable security return divided by one minus the client's tax rate to arrive at the correct answer to this problem. TEY= (Tax Exempt Security) ÷ (1 - the investor's marginal tax rate). This should include the state rate in the calculation of marginal rates as the municipal security is state tax-free in the state of issue if one is a resident of that state. = .05 ÷ (1 - .36 - .07) = .0877

A red herring is: A) An IP that is considered a "hot" offering. B) An underwriting that the SEC considers especially speculative. C) A prospectus for a public offering of securities by the current shareholders of a private company. D) A preliminary prospectus issued by the managing house of an offering.

D) A preliminary prospectus issued by the managing house of an offering. Rationale The correct answer is "D." The red herring is so called because of the red lettering notifying prospective investors of its status as a prospectus without prices included.

Which of the following describes an investor's position in purchasing a put and a call on the same security, at the same exercise price, for the same period of time? A) A spread. B) A strap. C) A strip. D) A straddle.

D) A straddle. Rationale The correct answer is "D." In a "spread", the prices are different. In a "strip", the price and time are the same, but two puts and one call are purchased. In a "strap", two calls and one put are purchased with price and time the same.

An investor in improved land (with an office building) is concerned most with which one of the following factors? A) Net income of investment. B) Reselling the property within three years. C) Real estate taxes. D) Cash flow expected to be generated by the property.

D) Cash flow expected to be generated by the property. Rationale The correct answer is "D." An office building is purchased to rent space; therefore, cash flow is of paramount importance. Net income without the information that leads up to this final figure is not as valuable as cash flow information. Resale, commissions and taxes are secondary concerns if the property is purchased (with an office building) for cash flow.

Which of the following is not an appropriate match? A) Classification by time: Spot markets. B) Classification by type of claim: Equity markets. C) Classification by participants: Mortgage markets. D) Classification by products: Money markets.

D) Classification by products: Money markets. Rationale The correct answer is "D." Money market securities are short-term instruments categorized by time considerations, not product. Look at this from the product to determine the classification. For example, money markets and spot markets are classified as according timing because they are either short term maturities or current price. The common component when classifying these type of securities is timing. Equity and debt markets can be classified as to the order of claims in the event of liquidation. "Type of claims" simply refers to debt vs. equity and which is more senior. Bond markets, which include mortgage bonds, are divided into short, intermediate and long term markets. Each market has participants that prefer different segments of the yield curve. A participant in this case is an insurance company, bank, manufacturing company, etc. Different participants will prefer mortgage bonds over shorter term maturities.

The primary difference between open-end and closed-end investment companies would be: A) Closed-end funds always sell at par value. B) Open-end funds do not charge sales fees. C) Closed-end funds guarantee the Net Asset Value (NAV) at the time of sale or purchase. D) Closed-end funds sell only a limited number of shares.

D) Closed-end funds sell only a limited number of shares. Rationale The correct answer is "D." Closed-end funds offer a limited number of shares, while open-end funds continually create new shares as new monies are obtained. Closed-end funds offer no price guarantees and do not always sell at net asset value.

Of the following investment, which is designed to provide growth and income? A) Raw land. B) Fixed premium annuity. C) Non-participating mortgage Real Estate Investment Trust (REIT.) D) Convertible bond.

D) Convertible bond. Rationale The correct answer is "D." Raw land may appreciate, but provides no income. A fixed premium annuity provides income, but no growth. Mortgage REITs offer income but no growth. Convertible bonds offer income as a bond and growth potential when converted to a stock.

An airline is considering issuing bonds to finance eight new airplanes that will be delivered in six months. Which type of bond will the airline issue? A) Income bond. B) Debenture. C) High-yield bond. D) Equipment trust certificate.

D) Equipment trust certificate. Rationale The correct answer is "D." Income bonds (Choice "A") are high risk bonds, usually issued by financially troubled firms. Debentures (Choice "B") are unsecured debt. High-yield bonds are lower quality than investment grade and cost the issuer more in interest payments. The airplanes would serve as collateral on the Equipment Trust Certificates.

The theory of the Yield Curve that attempts to explain the yield curve based upon future rates of inflation is the: A) Liquidity Preference Theory. B) Market Segmentation Theory. C) Short-Wave Theory D) Expectations Theory.

D) Expectations Theory. Rationale The correct answer is "D." Liquidity preference states that investors prefer liquidity, therefore, short-term money pays less. The market segmentation theory states that supply and demand explain at various maturities the shape of the yield curve.

As your client, Joe Stockhill has built an impressive portfolio of aggressive growth stocks. Recently, he expressed an interest in purchasing gold for his portfolio, but he does not fully understand the concepts surrounding its relationship to the stock market. How would you explain it to him? A) Due to its extremely volatile nature, gold should not be purchased in conjunction with an aggressive growth stock portfolio. B) Gold should only be purchased when inflation is low, otherwise one runs the risk of losing portfolio value. C) Gold has an inverse relationship to the market and should only be purchased when stock prices are rising. D) Gold has a negative correlation to the market and can be used when interest rates are rising as a hedge against inflation.

D) Gold has a negative correlation to the market and can be used when interest rates are rising as a hedge against inflation. Rationale The correct answer is "D." Gold's volatility has nothing to do with adding it or not adding it to a portfolio. Gold has tended to act as a hedge against inflation due to its negative correlation to the market. It is best purchased at the onset inflationary times, but a portfolio will not necessarily lose value by its purchase. Rising stock prices generally have meant stationary or decreasing gold prices.

Insured municipal bonds usually: A) Have higher rates of return because of the additional cost of insurance premiums. B) Guarantee the payment of interest on revenue bonds only. C) Are subject to review and an annual renewal of the insurance. D) Have lower rates of return because they have lower risk.

D) Have lower rates of return because they have lower risk. Rationale The correct answer is "D." Because of the protection afforded by the bond's insurance, the municipality issuing the bond will pay a lower return as a result of the reduced risk.

Which of the following statements about call options and warrants are true? I. Call options are created by individuals; warrants are issued by corporations. II. Call options generally have shorter durations than warrants. III. When an investor exercises either a call option or a warrant, he or she receives the stock shares from a writer of the option or warrant. IV. The call option or warrant writer uses this strategy to increase investment income from relatively stable underlying stocks. A) I and II only. B) II and III only. C) II and IV only. D) I, II and III only.

D) I, II and III only. Rationale The correct answer is "D." Choice "IV" - An option premium is paid to the writer upon purchase, where as a warrant does not increase company profit until it is exercised.

Mortgage-backed securities may contain which of the following risks: I. Purchasing power risk. II. Interest rate risk. III. Prepayment risk. A) II only. B) I and II only. C) I and III only. D) I, II and III.

D) I, II and III. Rationale The correct answer is "D." If rates on the mortgage backed securities do not keep up with inflation and rising rates, their purchasing power will be reduced, as will their value (interest rate risk). If interest rates fall, the mortgagees may seek refinancing and prepay their obligations early.

Physical assets might be suitable as an investment in the portfolio of an investor looking for: A) Deflationary hedges. B) Stability of periodic cash flows. C) Short-term investments. D) Long-term capital gains.

D) Long-term capital gains. Rationale The correct answer is "D." Hard assets are generally considered a hedge against inflation, which will lead to price appreciation and potential capital gains.

Eddie Bauer bought a tax-exempt Original Issue Discount (OID) bond in November of 1998. Which of the following statements is/are true? I. The bond basis increases at a set rate each year. II. The difference between maturity value and the original issue discount price is known as the OID. III. The bond's earnings are treated as exempt interest income. IV. The bond was issued at a discount to its par value. A) II and III only. B) I and IV only. C) I, II and IV only. D) I, II, III and IV.

D) I, II, III and IV. Rationale The correct answer is "D." All of the above statements are descriptions of the Original Issue Discount bond.

Which of the following are requirements as part of the due diligence process where securities investments are concerned? I. Company management. II. Financial track record of the firm. III. Company status. IV. Company stability. V. Accounting procedures used. A) I and II only. B) III and IV only. C) I, II, IV and V only. D) I, II, III, IV and V.

D) I, II, III, IV and V. Rationale The correct answer is "D." All of the above are part of the due diligence process. This is by no means a complete listing. Company status is public or privately held.

Fundamental analysis includes which of the following? I. Debt as a percent of total capital. II. A 39 week moving average of a company's stock prices. III. Interest rate trends. IV. Growth rate of the industry of which a company is a part. A) I and II only. B) I and III only. C) II and IV only. D) I, III and IV only.

D) I, III and IV only. Rationale The correct answer is "D." Choice "II" is a tool used by technicians to predict future prices. All the others choices are part of fundamental analysis.

Jack Rich has an investment portfolio equally divided among the following funds: Energy sector fund, Bond Unit Investment Trust (25-year average maturity), and a Money Market fund. He is a buy-and-hold investor. Which of the following risks is his portfolio exposed? I. Business risk. II. Interest rate risk. III. Political risk IV. Purchasing power risk. A) I and III only. B) II and IV only. C) I, II and III only. D) I, III and IV only.

D) I, III and IV only. Rationale The correct answer is "D." Interest rate risk does not affect a bond investor if he or she holds the securities to maturity. This is how unit investment trusts are structured. The energy sector will be directly impacted by regulatory influences of a political nature.

Which of the following are characteristics of the payout ratio? I. The percentage of compensation paid to the top 10 executives of a company as a percentage of net income. II. The percentage of net income paid out as dividends. III. A measure of a company's earnings retention philosophy. IV. A measure of a company's attitude toward compensation of top executives on the basis of performance. A) I only. B) II only. C) I and IV only. D) II and III only.

D) II and III only. Rationale The correct answer is "D." The dividend payout ratio is the portion of earnings which a company pays its investors. The balance of earning retained by the company comprises its retention ratio. The dividend payout ratio is: Dividend Per Share ÷ Earnings Per Share

A new client owns a U.S. Treasury bond that matures in 26 years. She purchased the bond because she was told that Treasury bonds are risk free. Which of the following statements about Treasury security risks should you communicate to your client? I. Treasury securities do not have interest rate risk because their coupons are fixed at the time of issue. II. Treasury securities with long maturities have purchasing power risk because their coupon returns are fixed, even if inflation rates rise substantially over the holding period. III. Treasury securities have country risk because they are direct obligations of a national government. IV. Treasury securities do not have default risk because the federal government has the power to tax and create money. A) I and II only. B) I and III only. C) II and III only. D) II and IV only.

D) II and IV only. Rationale The correct answer is "D." Choice "I" - Treasury securities have interest rate risk if sold prior to maturity. Choice "III" - U.S. government issued securities are said to be free of political or "country" risk.

Lily Wassenbaum asks for your assistance in designing an educational investment program for her eight-year-old son, Max. She expects to need the funds in about 15 years when her AGI will be approximately $55,000. She wants to invest at least part of the funds in tax-exempt securities. Help her select which investment(s) would yield tax-exempt interest on her federal return if the proceeds were used to finance Max's education. I. Treasury bills. II. EE bonds. III. GNMA funds. IV. Zero coupon Treasury bonds. A) III and IV only. B) I, III and IV only. C) II and III only. D) II only.

D) II only. Rationale The correct answer is "D." Options "I", "III" and "IV" all generate taxable income during their inclusion in Lily's portfolio: T-bills upon maturity, GNMA when income is paid, and Treasury zeros as the interest accrues within the bond, even though Lily will see no return paid to her. This is known as phantom income and it is fully taxable.

Which of the following are advantages of dividend reinvestment plans? I. Reinvested dividends currently are tax deferred. II. They help firms raise new capital. III. They give investors a systematic way to accumulate capital. IV. Companies build goodwill by offering these plans to shareholders. A) I and II only. B) II and IV only. C) III and IV only. D) II, III and IV only.

D) II, III and IV only. Rationale The correct answer is "D." All are true with the exception of dividends paid on a mutual fund are taxable, even when reinvested.

Bob and Betty have approached you looking for the right hedge against possible, expected future inflation. You suggest to them that they: A) Invest in technology stocks. B) Invest in commodity futures. C) Invest in long-term U.S. Treasury issues. D) Invest in precious metals.

D) Invest in precious metals. Rationale The correct answer is "D." None of the choices are necessarily stellar, but in contrast to the other choices, Option "D" makes far more sense, as metals have generally performed well as inflation hedges over time.

Which of the following statements concerning the S&P 500 is incorrect? A) It has less dramatic fluctuations than the Dow Jones Industrial Average. B) It is a reflection of broad sectors of the market. C) It is a value-weighted index. D) It is a broader base measure of the stock market than the Wilshire 5000 Index.

D) It is a broader base measure of the stock market than the Wilshire 5000 Index. Rationale The correct answer is "D." The DJIA tracks 30 stocks, the S&P 500 tracks 500 stocks, and the Wilshire 5,000 tracks slightly more than 6,500 stocks. The broader the stock base (or larger number of stocks), the less the fluctuation of the index.

Mark is the 100% owner of Widget Manufacturing, Inc. (WMI). The WMI 401(k) plan covers 35 employees with 70% of the employees under age 40. Employee turnover is high. Mark wants to retire in 20 years at age 65. Mark has a medium tolerance for volatility within his investments. The market value of the 401(k) is $4,000,000 and Mark is the trustee and manages the investments. The portfolio consists of the following assets: - 10% short-term CDs with staggered maturity date. - 20% limited partnership interest in a private commercial real estate project which generates a high income yield. - 40% in a brokerage account invested in four stocks. - 30% in long-term government bonds with staggered maturity dates. You have been retained to evaluate the appropriateness of the portfolio. Which of the following statements best describes the portfolio? I. Short-term certificates of deposit have a fixed maturity date and therefore are not appropriate for liquidity purposes. II. The brokerage account investments are not adequately diversified. III. The investment in the limited partnership may be subject to unrelated business taxable income and therefore is inappropriate. IV. Overall, the asset classes selected by the plan are sufficiently diversified and therefore minimize overall portfolio volatility. A) I and II only. B) II and III only. C) I, II and III only. D) II, III and IV only.

D) II, III and IV only. Rationale The correct answer is "D." Statement "I" - Short-term CDs are an appropriate choice for liquidity needs due to fixed value of the vehicle and short maturity periods. Statement "IV" - Even though the current investments within the asset classes are not properly positioned, the asset class allocation would, under Modern Portfolio Theory, reduce overall volatility to the plan.

Which of the following are characteristics of Government National Mortgage Association (GNMA) securities? I. Investors are guaranteed, by the U.S. government, against losses arising from investments in GMNA securities. II. The amount received by the investor each month may vary due to prepayment by homeowners. III. The realized yield on the certificates can be somewhat variable because of the principal prepayments. IV. If mortgage rates decrease, prepayments may increase. A) I and IV only. B) II and IV only. C) I, II, and III only. D) II, III, and IV only.

D) II, III, and IV only. Rationale The correct answer is "D." GNMA is "on budget" agency debt. This means the pools of mortgages are backed by the full faith, credit, and taxing power of the U.S. government itself. The government backs the issue against default, NOT against investor loss through poor timing or poor choices. It should also be noted no U.S. government agency debt has ever defaulted.

Growth and income mutual funds have which of the following characteristics? I. They usually hold both over-the-counter stocks and stock warrants. II. They provide tax-free income to the investor. III. They usually have a dual objective to provide both capital appreciation and current income. IV. They may own convertible securities, growth stocks, and income stocks. A) I only. B) I and II only. C) I and IV only. D) III and IV only.

D) III and IV only. Rationale The correct answer is "D." Growth and income mutual funds hold stocks and bonds. They do not provide tax free income to investors unless they contain some municipal bonds.

Of the pricing models we have examined, the following use "additional" factors to measure impact on return and arrive at appropriate portfolio returns: I. CAPM II. Black's Zero-Beta CAPM III. Merton's Multi-factor CAPM IV. Arbitrage Pricing Theory A) I only. B) I and II only. C) III only. D) III and IV only.

D) III and IV only. Rationale The correct answer is "D." Only Merton's Multi-factor CAPM and Ross' Arbitrage pricing theory include additional factors beyond standard rates found in the CAPM.

Several individual investments each have high standard deviations. Which of the following are true about the standard deviation for a portfolio of these same investments? I. Has to be high since the standard deviations are high. II. Has to be low since the standard deviations are high. III. Can be low if there is a low correlation of returns between the investments. IV. Can be high if there is a high correlation of returns between the investments. A) I and III only. B) I and IV only. C) II and III only. D) III and IV only.

D) III and IV only. Rationale The correct answer is "D." The relationship between correlations and standard deviation are direct in nature. A high correlation means securities will move in the same direction. High standard deviation with high correlation means high risk. Low correlation means that even the impact of high deviation securities can be reduced.

American Depository Receipts (ADRs) are for the following purpose(s): I. Finance foreign exports. II. Eliminate currency risk. III. Sell U.S. Securities in overseas markets. IV. Trade foreign securities in U.S. markets. A) I and III only. B) I and IV only. C) II and IV only. D) IV only.

D) IV only. Rationale The correct answer is "D." ADRs provide an opportunity for Americans to purchase foreign securities.

Modern "asset allocation" is based upon the portfolio theory model developed by Markowitz. Of the following statements, which correctly identify the model? I. Negatively correlated assets are necessary to reduce the risk of portfolios. II. In creating a portfolio, diversifying across asset types (e.g., stocks and bonds) is less effective than diversifying within an asset type. III. The efficient frontier is relatively insensitive to the input variable. IV. The risk, return and covariance of assets are important input variables in creating portfolios. A) I and II only. B) I, II and III only. C) III only. D) IV only.

D) IV only. Rationale The correct answer is "D." Option "I" is not correct. Although negative correlation of assets reduces risk, it is not NECESSARY that assets be negatively correlated. As long as assets are not perfectly positively correlated, risk will be reduced. Option "II" is also a false statement, diversifying across asset types is MORE effective. Option "III" is incorrect in that the efficient frontier IS sensitive to input variables.

The cumulative feature on a preferred stock is best described in the following: A) The preferred stock gets to cast its entire total of votes in a grouping for one seat on the board of directors if the shareholders so desire. B) The preferred shareholder has the option of accruing a certain number of shares and then converting them to common stock. C) If there are additional or extra dividends declared, the preferred shareholders have the right to share in the profits. D) If dividends are not paid in a given cycle, they cannot be paid to anyone else until they are paid to preferred shareholders.

D) If dividends are not paid in a given cycle, they cannot be paid to anyone else until they are paid to preferred shareholders. Rationale The correct answer is "D." Preferred stocks are non-voting shares. The description of Option "A" is of common stocks' cumulative voting rights. Option "B" refers to convertibility, while Option "C" addresses participating preferred stocks.

Whenever there is a cash dividend issued on an underlying stock, the price (or premium) for a call option available on that stock tend to be: A) Unaffected. B) Higher. C) Volatile. D) Lower.

D) Lower. Rationale The correct answer is "D." Cash dividends will generally tend to drive the price of the underlying security lower and along with it, the call option prices.

Which portfolio manager has out performed the market, if the market had a standard deviation of 18.27%, a return of 10.23%, and the risk-free rate was 2.76% over the period in consideration? A) Manager A, over the period under consideration, had a portfolio which returned 12.12% with a standard deviation of 23.51% B) Manager B, over the period under consideration, had a portfolio which returned 13.3% with a beta of 1.44 C) Manager C, over the period under consideration, had a portfolio which returned 9.23% with a beta of .89 D) Manager D, over the period under consideration, had a portfolio which returned 15.12% with a standard deviation of 29.85%

D) Manager D, over the period under consideration, had a portfolio which returned 15.12% with a standard deviation of 29.85% Rationale The correct answer is "D." Keep in mind we want the Treynor or Sharpe indexes used here and the beta of the market is one. With this in mind, we must first calculate the value of the indexes for the market in both the Treynor measure and the Sharpe measure. This being done, we calculate each portfolio for either Treynor Index (if they use beta) or Sharpe Index (if they use standard deviation as a measure of risk). Only manager D beat the calculated market figures. Market Sharpe = 10.23 - 2.76 18.27 = 0.4088 Treynor = 10.23-2.76 1 = 7.47 A Sharpe = 12.12-2.76 23.51 = 0.3981 Treynor = No Beta Provided B Sharpe = No standard deviation provided Treynor = 13.3-2.76 1.44 = 7.3194 C Sharpe = No standard deviation provided Treynor = 9.23-2.76 0.89 = 7.2696 D Sharpe = 15.12-2.76 29.85 = 0.414 Treynor = No Beta Provided

To immunize a bond portfolio over a specific investment horizon, an investor would do which of the following? A) Match the maturity of each bond to the investment horizon. B) Match the duration of each bond to the investment horizon. C) Match the average weighted maturity of the portfolio to the investment horizon. D) Match the average weighted duration of the bond portfolio to the investment horizon.

D) Match the average weighted duration of the bond portfolio to the investment horizon. Rationale The correct answer is "D." Duration, not maturity is used to immunize a portfolio. The average weighted duration rather than the duration of each specific bond is used for successful portfolio immunization.

You are currently reviewing Shanda's Roth IRA investment portfolio. She is 35 years of age with a moderate risk tolerance. She is single and has no children. Which of the following investments are you likely to recommend she remove from the existing portfolio? A) Large Cap Mutual Fund B) International Mutual Fund C) Corporate Bond Mutual Fund D) Municipal Bond Mutual Fund

D) Municipal Bond Mutual Fund Rationale The correct answer is "D." Municipal bonds are not generally suitable investments in a Roth IRA because of the tax deferral that is already built into the Roth IRA and the potential for tax free distributions if you meet the qualifying rules.

Match the investment characteristic(s) listed below which describe(s) an open-end investment company. A) Only passive management of the portfolios. B) Shares of the fund are normally traded in major secondary markets. C) Both "A" and "B." D) Neither "A" nor "B."

D) Neither "A" nor "B." Rationale The correct answer is "D." Option "A" is incorrect because open-end funds are both passively and actively managed. Option "B" is incorrect because open-end fund shares are traded directly with the fund, not on the secondary market.

Use the following information to answer the question: Stock Beta ERR Amt Invested A 1.4 15% $10,000 B 1.2 12% $15,000 C 0.9 9% $11,000 Your client has expressed a desire to reduce the risk in her portfolio without reducing the expected rate of return. Your recommendation would be as follows: I. Increase the amount of stock A and decrease the amount of stock C. II. Increase the amount of stock B and decrease the amount of stock C. III. Increase the amount of stock C and decrease the amount of stock A. A) III only. B) I and II only. C) I and III only. D) None of the above.

D) None of the above. Rationale The correct answer is "D." The risk in this portfolio cannot be reduced without a commensurate reduction in return based on the information given. To reduce risk, you need a lower weighted average beta, than the current portfolio allocation. To accomplish this, you must increase the amount invested in C (or maybe B too), but the point is you are increasing the amount invested in a security with a lower expected return thereby decreasing the expected return of the portfolio.

Put option sellers do best if the market price of the stock: A) Falls. B) Rises. C) Falls or remains at the same price. D) Rises or remains at the same price.

D) Rises or remains at the same price. Rationale The correct answer is "D." The put option seller looks for the security price to rise and opposites the position of the put buyer who looks for a falling security price. If the price rises or stays the same, the put seller keeps the premium.

Which of the following option strategies would be considered the most risk? A) Buying a call. B) Buying a put. C) Selling a covered call. D) Selling a put.

D) Selling a put. Rationale The correct answer is "D." Buying a put or call option limits the investor's loss to the premium paid. With a covered call, the investor owns the underlying stock, which offsets any loss associated with selling the call. Selling a put is the most risky of the strategies listed because the stock could fall to zero.

Your client has asked you to assist her in examining possible additions to her bond portfolio. She has expressed a desire to minimize risk at this stage in her planning process, and to assure income beginning at the point of her retirement, and lasting throughout. She has a tentative retirement date in seven years at age 65. She will then have an eighteen year life expectancy. Which of the following is an appropriate addition to her current portfolio? I. 25-year AAA-rated corporate bonds with a seven-year maturity. II. 20 year AAA-rated municipal bonds with a seven-year duration. III. 25-year AAA-rated corporate zeroes with a seven-year duration. IV. 20-year US Treasury zeroes with a seven-year maturity. V. 25-year AAA-rated corporate bonds with a seven-year duration. A) I, III and V only. B) II, III and V only. C) III and IV only. D) V only.

D) V only. Rationale The correct answer is "D." The client is looking for income to begin in 7 years. Therefore anything maturing in 7 years will not provide that income. Zeroes provide no income. She wants something out 25 years, not 20 years. Thus, option "V" is the only appropriate answer.

Your client is considering the two stocks described below. Assume for this question that the risk-free rate is 6%, the expected return on the market is 14%, and the market's standard deviation is 18%. Stock A price per share = $18 Stock A annual dividend = $2 Stock A dividend growth rate = 3% Stock A Beta = 1.1 Stock A standard deviation = 21% Stock A realized return over past 12 months = 15% Stock B price per share = $12 Stock B annual dividend = $1.50 Stock B dividend growth rate = 4% Stock B beta = 0.88 Stock B standard deviation = 14% Stock B realized return over past 12 months = 12.5% Which stock would you recommend your client purchase, and why? A) Stock A, because its intrinsic value is greater than Stock B's. B) Stock A, because its required return is greater than Stock B's. C) Stock A, because its risk-adjusted return is greater than Stock B's. D) Stock B, because it is selling for less than its intrinsic value.

D) Stock B, because it is selling for less than its intrinsic value. Rationale The correct answer is "D." Though there is a great deal of information here, one should be looking at the intrinsic values of the stocks and comparing them to the market prices of the stocks for over valuation or under valuation. A market selling price above intrinsic value is overvalued. A market price below the intrinsic value means the stock is undervalued and may be considered for purchase. The first step in this problem is to compute the required rate of return using the CAPM formula: R = Rf + B * (Rm - Rf) Stock A =14.8% = 6 + 1.1 * (14-6) Stock B = 13.04% = 6 + .88 * (14-6) Now that we have the required rate of return on the securities based on their volatility, I would utilize the Gordon Growth Model (or dividend model) to determine the price that the security should be at to be "fairly valued." V = D * (1 + g) / (R - g) Stock A =17.45 = [2 * (1+.03)] / (0.148-.03) Stock B = 17.25 = [1.5 * (1+.04)] / (0.1304-.04) So the "fair value" for our investor is $17.45 for stock A and it is trading at $18. Therefore the stock is overvalued (trading higher than its value). Stock B is valued at $17.25 for our investor and is trading at $12. Therefore the stock is undervalued (trading lower than its value).

You are about to choose a new mutual fund to add to client portfolios. As you review the Morningstar reports for the funds you are considering, you have focused on each fund's alpha as reported by Morningstar. Alpha tells you: A) Each fund's performance relative to a benchmark, such as the S&P 500. B) A fund's percentage return above the risk-free rate of return. C) By what percentage a fund's capital appreciation exceeded the capital appreciation of the average fund in its asset class. D) The difference between a fund's realized return and its risk-adjusted expected return.

D) The difference between a fund's realized return and its risk-adjusted expected return. Rationale The correct answer is "D." The alpha of a security may be calculated using the Jensen Model. The Jensen formula is on the formula sheet included with the CFP® exam materials. Alpha is the fund's actual return minus the risk adjusted expected return, as measured by CAPM.

According to fundamental analysis, which phrase best defines the intrinsic value of a share of common stock? A) The par value of the common stock. B) The book value of the common stock. C) The liquidating value of the firm on a per share basis. D) The discounted value of all future dividends

D) The discounted value of all future dividends Rationale The correct answer is "D." Intrinsic value is the discounted value of a future stream of cash flows. In the case of a stock, its dividends.

The market where exchange and broker dealer services are eliminated entirely is: A) The primary market. B) The secondary market. C) The third market. D) The fourth market.

D) The fourth market. Rationale The correct answer is "D." The fourth market is the market where corporation and institutional investors deal directly with one another. Primary market is where investment bankers and corporations meet to arrange offerings to the public. Secondary markets are where previously issued securities are sold (exchanges, etc.).

On the Markowitz Model, at the point of tangency, we have attained: A) The efficient frontier. B) The indifference curve. C) The unattainable. D) The optimal portfolio.

D) The optimal portfolio. Rationale The correct answer is "D." Option "A" - The efficient frontier measures what portfolios are attainable and which are unattainable. Option "B" - The indifference curve measures what level of risk an investor will accept for given levels of return.

Company A has 60% debt and 40% equity; Company B has 20% debt and 80% equity. Assume both companies have the same dollar amount of assets and net income before interest and taxes. Which one of the following statements is true? A) The unsystematic risk for the two companies is about equal. B) Company A's tax obligation will exceed Company B's. C) The company with the higher return on equity should be purchased by a risk-averse investor. D) The return on equity for Company A can be expected to exceed the return on equity for Company B. Rationale The correct answer is "D." Company A has a smaller amount of its assets financed by equity, therefore, with the same earnings in net income as Company B, the level of return on the equity of Company A would be greater. Purely for EXAMPLE (net income is provided, not calculated): A: Assets $10M; Liabilities $6M; Equity $4M; EBIDTA = $1M; I/Y = 5% Net Income = $700K; ROE = $700,000 ÷ $4M = 17.5% B: Assets $10M; Liabilities $2M; Equity $8M; EBIDTA = $1M; I/Y = 5% Net Income = $900K; ROE = $900,000 ÷ $8M = 11.25%

D) The return on equity for Company A can be expected to exceed the return on equity for Company B. Rationale The correct answer is "D." Company A has a smaller amount of its assets financed by equity, therefore, with the same earnings in net income as Company B, the level of return on the equity of Company A would be greater. Purely for EXAMPLE (net income is provided, not calculated): A: Assets $10M; Liabilities $6M; Equity $4M; EBIDTA = $1M; I/Y = 5% Net Income = $700K; ROE = $700,000 ÷ $4M = 17.5% B: Assets $10M; Liabilities $2M; Equity $8M; EBIDTA = $1M; I/Y = 5% Net Income = $900K; ROE = $900,000 ÷ $8M = 11.25% Or, if NI=700k, ROE= 8.75%


Ensembles d'études connexes

AP2 Reproduction Menstrual Cycle Steele Buie

View Set

Human A&P Terms for the FINAL : Be able to define or describe each of the following. I will randomly choose 50 of these in which you will need to define

View Set

Control - Working with control charts

View Set

Series 63 Chapter 1 Continued Further

View Set

NURS intervention EXAM 2 PrepU & lecture quizzes

View Set